february 2016 pennsylvania bar … 2016 pennsylvania bar examination essay questions and...

79
FEBRUARY 2016 PENNSYLVANIA BAR EXAMINATION Essay Questions and Examiners’ Analyses and Performance Test Pennsylvania Board of Law Examiners 601 Commonwealth Avenue, Suite 3600 P.O. Box 62535 Harrisburg, PA 17106-2535 (717) 231-3350 www.pabarexam.org ©2016 Pennsylvania Board of Law Examiners

Upload: truongcong

Post on 19-Mar-2018

228 views

Category:

Documents


6 download

TRANSCRIPT

Page 1: FEBRUARY 2016 PENNSYLVANIA BAR … 2016 PENNSYLVANIA BAR EXAMINATION Essay Questions and Examiners’ Analyses and Performance Test Pennsylvania Board of Law Examiners 601 Commonwealth

FEBRUARY 2016

PENNSYLVANIA BAR

EXAMINATION

Essay Questions and Examiners’ Analyses

and

Performance Test

Pennsylvania Board of Law Examiners

601 Commonwealth Avenue, Suite 3600

P.O. Box 62535

Harrisburg, PA 17106-2535

(717) 231-3350

www.pabarexam.org

©2016 Pennsylvania Board of Law Examiners

Page 2: FEBRUARY 2016 PENNSYLVANIA BAR … 2016 PENNSYLVANIA BAR EXAMINATION Essay Questions and Examiners’ Analyses and Performance Test Pennsylvania Board of Law Examiners 601 Commonwealth

Table of Contents

Index ..................................................................................................................................................ii

Question No. 1: Facts and Interrogatories, Examiner's Analysis and Grading Guidelines .............. 1

Question No. 2: Facts and Interrogatories, Examiner's Analysis and Grading Guidelines ..............10

Question No. 3: Facts and Interrogatories, Examiner's Analysis and Grading Guidelines ..............20

Question No. 4: Facts and Interrogatories, Examiner's Analysis and Grading Guidelines ..............27

Question No. 5: Facts and Interrogatories, Examiner's Analysis and Grading Guidelines ..............39

Question No. 6: Facts and Interrogatories, Examiner's Analysis and Grading Guidelines ..............48

Performance Test and Grading Guidelines ........................................................................................56

i

Page 3: FEBRUARY 2016 PENNSYLVANIA BAR … 2016 PENNSYLVANIA BAR EXAMINATION Essay Questions and Examiners’ Analyses and Performance Test Pennsylvania Board of Law Examiners 601 Commonwealth

Index

Question No. 1

1. Decedents’ Estates: advancement

2. Decedents’ Estates: intestacy

3. Professional Responsibility: conflict of interest

4. Federal Income Tax: involuntary conversion

Question No. 2

1. Torts: negligence

2. Civil Procedure: joinder

3. Evidence: offer to pay medical expenses, compromise and settlement

4. Civil Procedure: discovery- medical information

Question No. 3

1. Criminal Law: robbery

2. Criminal Law: miranda warnings

3. Evidence: hearsay – prior testimony

4. Family Law: special relief – court equitable power

Question No. 4

1. Constitutional Law: commercial speech

2. Employment Law: discrimination: *after discovered evidence

3. Civil Procedure: venue

ii

Page 4: FEBRUARY 2016 PENNSYLVANIA BAR … 2016 PENNSYLVANIA BAR EXAMINATION Essay Questions and Examiners’ Analyses and Performance Test Pennsylvania Board of Law Examiners 601 Commonwealth

Question No. 5

1(a) & (b). Property: fee simple determinable with possibility of reverter

2. Contracts: frustration of purpose

3(a). Property: constructive eviction

3(b). Property: implied warranty of habitability landlord/tenant

4. Property: tenancy by Entireties, tenancy in common

Question No. 6

1(a). Corporations: fiduciary duty, corporate opportunity

1(b). Corporations: remedy breach of fiduciary duty

2. U.C.C. Art. III: implied warranty merchantability, implied warranty fitness for particular

purpose

3. Conflict of Law: situs rule

iii

Page 5: FEBRUARY 2016 PENNSYLVANIA BAR … 2016 PENNSYLVANIA BAR EXAMINATION Essay Questions and Examiners’ Analyses and Performance Test Pennsylvania Board of Law Examiners 601 Commonwealth

Question No. 1

Angela was a retired widow living in E County, Pennsylvania. In November 2014, she

wanted to make a will. Angela’s only child, David, referred her to Larry, an E County estate

lawyer, to draft her will. David is married to Jane, and they have three adult children.

Angela was quite impressed by Larry, who agreed to meet her in her home. Angela

asked Larry if he could do other legal work for her. Larry replied, “It would have to be worth

my while. In your will you could give something to anyone.” Angela’s housekeeper overheard

the entire conversation. Angela paid Larry a reasonable fee to draft the will and then told Larry

he could add a bequest to himself in the will. She instructed Larry to name David as executor.

The distribution clause of the will stated:

I give my attorney Larry $10,000. All the remainder of my estate, real and

personal, I give to my son David.

The will was validly executed in December of 2014. Soon thereafter Angela became

very ill and died in March of 2015. She had not requested any other legal services from Larry

after the will was executed. Her will was admitted to probate with Larry acting as attorney.

Larry charged reasonable fees for handling the estate. Larry distributed the $10,000 to himself

and the remainder to David after all taxes and costs were paid in August 2015.

In October 2015, Michael, a son of David and Jane, turned 21. For Michael’s birthday,

David gave him the keys and signed title to a sports car valued at $25,000 at that time, which

David had inherited from Angela’s estate, saying, “I was going give this to you in my will when

I die, but you might as well enjoy it now.” Michael later wrote an email from his college

dormitory thanking David for the car and writing that he was glad David “was giving him part of

his inheritance now.” David printed the email and showed it to Jane. Jane kept the email.

1

Page 6: FEBRUARY 2016 PENNSYLVANIA BAR … 2016 PENNSYLVANIA BAR EXAMINATION Essay Questions and Examiners’ Analyses and Performance Test Pennsylvania Board of Law Examiners 601 Commonwealth

In November 2015, a home David and Jane had purchased together in 2005 with cash,

$75,000 of which was the value of the structure, was totally destroyed by fire. They had used it

as a primary residence for the first three years after purchasing the home, but it later became their

vacation home. They added no significant improvements to it, nor did they ever rent it out; they

kept it only for their personal use. The insurance on the home was based on the current structure

value of $200,000, and in December 2015 a check for that amount was issued jointly to David

and Jane. They deposited the check in a joint savings account.

In late December 2015, David died intestate. His net individual estate was in the high six

figures, including his inheritance from Angela. Jane was appointed Administrator of David’s

estate. Jane’s attorney, Mary, explained the law regarding intestate estates to Jane, and they

agreed that David’s estate would be distributed in accordance with Pennsylvania’s rules of

intestate distribution. Jane told Mary that David had given Michael the sports car. Jane then

showed Michael’s email to Mary. Jane asked if an adjustment could be made as a result of the

gift when calculating the value and intestate distribution of David’s estate.

1. As a result of the gift of the car by David to Michael could any adjustment be

made to the value or distribution of David’s estate?

2. Assume for this issue the car had not been gifted to Michael and was still a part of

David’s estate. How will David’s estate be distributed proportionately to his

intestate heir(s)?

3. Shortly after David’s death, Angela’s housekeeper called Jane to tell her about the

conversation she had overheard between Larry and Angela. Jane then asked Mary

about Larry’s $10,000 inheritance. Assuming all the facts stated above as true,

did Larry violate any of the Pennsylvania Rules of Professional Conduct in

connection with the bequest to himself in Angela’s will?

4. David and Jane regularly filed joint federal tax returns on a cash basis. Assuming

that Jane again files jointly, as is permitted by law for the year of a spouse’s

death, and that Jane chooses not to rebuild or replace the vacation home, what are

the 2015 federal income tax effects to David and Jane regarding the $200,000

insurance payment for the destruction of their vacation home?

2

Page 7: FEBRUARY 2016 PENNSYLVANIA BAR … 2016 PENNSYLVANIA BAR EXAMINATION Essay Questions and Examiners’ Analyses and Performance Test Pennsylvania Board of Law Examiners 601 Commonwealth

Question No. 1: Examiner’s Analysis

1. If Jane seeks an adjustment, the value of the car will likely be determined to be an

advancement on Michael’s share of David’s estate, and its value will be added to the

value of David’s intestate estate and deducted from Michael’s share.

The car will likely be considered an advancement against Michael’s share in David’s

estate. “An advancement . . . is an irrevocable gift by a parent to a child in anticipation of such

child’s future share of the parent’s estate.” Laughlin Estate, 354 Pa. 43, 47, 46 A.2d 477, 479

(1946) (further citation omitted). The Pennsylvania Probate Estates and Fiduciaries (PEF) Code

states, in relevant part, the following:

If a person dies intestate as to all or any part of his estate, property

which he gave in his lifetime to an heir is treated as an

advancement against the latter’s share of the estate only if declared

in a writing by the decedent or acknowledged in writing by the heir

to be an advancement.

20 Pa. C.S. § 2109.1. This statute was enacted in 1976 as a change in previous law which

presumed such a substantial gift from a decedent to one of his children was an advancement.

The statute now requires a writing to designate the gift as an advancement. 20 Pa.C.S. § 2109.1,

Jt. St. Govt. Comm. Cmt. 1976; DANNY R. VEILLEUX, J.D., 32 Standard Pa. Practice 2nd § 154:11

(Thompson Reuters 2015) (citation omitted). The burden of proof is on the person asserting that

a gift was an advancement. In re Clark’s Estate, 303 Pa. 538, 544, 154 A. 919, 921 (1931); see

also VEILLEUX, supra 154.11. The statute does not state that a signature or other formality is

required to define “a writing” for its purposes. See 20 Pa.C.S. § 2109.1.

David died intestate. He was the sole owner of the car after it was distributed to him with

all the remainder of Angela’s estate in accordance with Angela’s will. He gave the car to his son

Michael on Michael’s 21st birthday stating that Michael “may as well enjoy [the car] now”

instead of receiving it as a bequest from David’s estate later. David’s statement would not alone

have had the effect of making the car an advancement on Michael’s inheritance. Jane has the

burden of proof to show that the car was an advancement. Mary should advise Jane that she

could assert that the car was an advancement on Michael’s share of David’s estate as Jane has

Michael’s email thanking David and repeating David’s statement to him that he might as well

enjoy the car now instead of by bequest in his will when David died. This email would be a

writing showing that the gift of the car to Michael was an advancement. Thus Michael

“acknowledged in writing” the intent of David to treat the gift of the car as something he would

have bequeathed to Michael had David died still owning it. Accordingly, Michael’s email to

David satisfied the requirements of the statute to make the gift of the car an advancement on

Michael’s share of David’s intestate estate.

3

Page 8: FEBRUARY 2016 PENNSYLVANIA BAR … 2016 PENNSYLVANIA BAR EXAMINATION Essay Questions and Examiners’ Analyses and Performance Test Pennsylvania Board of Law Examiners 601 Commonwealth

The PEF Code also provides that “the property advanced is valued as of the time the heir

came into possession or enjoyment of the property or as of the time of the death of the decedent,

whichever first occurs.” 20 Pa.C.S. § 2109.1. The facts provide that $25,000 was the value of

the car when David gave it to Michael. The $25,000 value is then added to the value of David’s

solely owned assets in his estate; increasing the amount of his estate by that amount. See

Laughlin Estate, 354 Pa. at 48 46 A.2d at 479, citing inter alia, McConomy’s Estate 170 Pa. 140,

32 A. 608 (1895). The concept of adding the value of an advancement to an estate for

distribution is known as “hotchpot,” which is defined at length in Laughlin Estate as follows:

Advancements are to be treated as if repayment had been made to

the estate, the total divided among the heirs or beneficiaries, and

the advancements deducted from the shares of those advanced.

The whole is placed in hotchpot, the advancements added and the

total divided . . . . The doctrine of hotchpot is of ancient origin. . . .

It constitutes a blending or throwing into a common lot or stock of

property for equality of division. . . . The doctrine rests on the

presumed desire of an ancestor to equalize the estate among his

heirs, not only as to the property left at the time of his death, but as

to all property that came from him, so that one child shall not be

preferred to another child in the final settlement of his estate.

Laughlin Estate, 354 Pa. at 48-49, 46 A.2d at 479-80 (citations omitted).

Accordingly, the value of David’s estate would be increased by $25,000, and Michael’s

share of the increased estate value would be reduced by the $25,000 value of the car.

2. David’s estate will be distributed to his spouse, Jane, and their three children. Jane

will receive the first $30,000 of the estate and 50% of the remaining estate, and the

three children will split the other half in equal 1/3 shares.

Where the decedent dies intestate and has surviving issue “all of whom are issue of the

surviving spouse also, the [surviving spouse receives] the first $30,000 plus one-half of the

balance of the intestate estate.” 20 Pa. C.S. § 2102. The facts provide that David died without

having made a will and that Jane and Mary determined that David’s estate would be distributed

in accordance with Pennsylvania rules of intestate succession. David’s estate was, as provided in

the question, in the high six figures. Accordingly, Jane would take an initial $30,000 and one-

half of the remaining estate value.

The remaining half of David’s estate would then go to David’s issue, his three children,

under the PEF Code. The PEF Code provides, in relevant part, that the remaining part of the

estate after the spouse takes her share shall pass “[t]o the issue of the decedent.” 23 Pa. C.S. §

2103. In turn “[t]he part of the estate passing to [those other than the decedent’s spouse] shall be

divided into as many equal shares as there shall be persons in the nearest degree of consanguinity

to the decedent living and taking shares therein.” 23 Pa.C.S. § 2104. As the facts state that

4

Page 9: FEBRUARY 2016 PENNSYLVANIA BAR … 2016 PENNSYLVANIA BAR EXAMINATION Essay Questions and Examiners’ Analyses and Performance Test Pennsylvania Board of Law Examiners 601 Commonwealth

David had three children, the estate would be divided evenly between them. Thus, each child

would be entitled to one-third of the remaining estate after Jane took her share.

3. Larry’s action in drafting Angela’s will with a $10,000 testamentary bequest to

himself is likely to be determined a violation of the Pennsylvania Rules of

Professional Conduct as it is a substantial gift and Larry is not related to Angela.

The bequest of $10,000 to Larry in Angela’s will that Larry drafted after an initial

consultation with Angela, presents an issue of professional responsibility regarding the scope

and magnitude of a lawyer obtaining benefits other than reasonable fees and costs from a client.

The Pennsylvania Rules of Professional Conduct (RPC) define a number of prohibitions or

cautions for lawyers engaging in various situations with the potential for conflict. See Pa.R.P.C.

Nos. 1.0 et seq. The RPC define conflicts of interest regarding gifts to attorneys from clients, in

relevant part, as follows:

A lawyer shall not solicit any substantial gift from a client,

including a testamentary gift, or prepare on behalf of a client an

instrument giving the lawyer or a person related to the lawyer any

substantial gift unless the lawyer or other recipient of the gift is

related to the client. For purposes of this paragraph, related

persons include a spouse, child, grandchild, parent, grandparent or

other relative or individual with whom the lawyer or the client

maintains a close familial relationship.

Pa.R.P.C. No. 1.8(c).

This Rule prohibits the preparation of “an instrument giving the lawyer . . . any

substantial gift” unless the lawyer or other recipient is related to the client, regardless of the

absence of solicitation. Here the facts show that Larry drafted Angela’s will, which contained a

bequest to him of $10,000. Thus he prepared the instrument giving himself a substantial gift in

violation of Rule 1.8(c).

Comment 6 to RPC 1.8 discusses the concept of gifts from clients to lawyers and the threshold of

a “substantial” unsolicited gift from a client which could be a violation for a non-related lawyer:

A lawyer may accept a gift from a client, if the transaction meets

general standards of fairness. For example, a simple gift such as a

present given at a holiday or as a token of appreciation is

permitted. If a client offers the lawyer a more substantial gift,

paragraph (c) does not prohibit the lawyer from accepting it,

although such a gift may be voidable by the client under the

doctrine of undue influence, which treats client gifts as

presumptively fraudulent. In any event, due to concerns about

overreaching and imposition on clients, a lawyer may not suggest

that a substantial gift be made to the lawyer or for the lawyer’s

5

Page 10: FEBRUARY 2016 PENNSYLVANIA BAR … 2016 PENNSYLVANIA BAR EXAMINATION Essay Questions and Examiners’ Analyses and Performance Test Pennsylvania Board of Law Examiners 601 Commonwealth

benefit, except where the lawyer is related to the client as set forth

in paragraph (c).

Pa.R.P.C. No. 1.8, cmt. 6.

The $10,000 written into Angela’s will as a bequest to Larry is certainly “substantial” and

is not a “simple gift.” It was not a gift given at a holiday or as a token of appreciation as it

appears that Larry suggested the bequest. Additionally, Larry cannot claim that the bequest was

intended to be part of or in lieu of compensation for his services as attorney for Angela’s estate

as the facts state he took fees for all of his services, and Angela had not required any services

from Larry after he had drafted her will. Thus the $10,000 is both substantial and a gift and it

was a violation of Rule 1.8 for Larry to draft a will for Angela that included this bequest.

Another comment to Rule 1.8 provides the following:

If effectuation of a substantial gift requires preparing a legal

instrument such as a will or conveyance, the client should have the

detached advice that another lawyer can provide. The sole

exception to this Rule is where the lawyer is a relative of the

donee.

Id. at cmt. 7.

In the facts provided, Larry is not a “related person” as Rule 1.8 (c) defines the term. He

had only recently been referred to Angela by her son David to draft her will. Comment 7

provides that in the event that a “substantial gift requires . . . a will . . . the client should have the

detached advice that another lawyer can provide.” Id. In this case, Larry did not ask or

recommend that another lawyer be consulted to provide detached advice to Angela. He simply

drafted the will giving himself $10,000. Thus Larry violated rule 1.8 of the RPC.

Moreover, the facts suggest that Larry “solicited” the gift. Angela’s housekeeper could

testify to the conversation in which, in response to Angela’s request that Larry do additional

work for her, Larry stated, “It would have to be worth my while. In your will you could give

something to anyone.” It was at that point that Angela told Larry to put a “gift” for himself in

her will. Thus, it appears under the facts that Larry solicited the substantial gift from Angela.

However, even if this testimony and evidence is not conclusive to show that Larry “solicited” the

$10,000 gift, Larry would still have a problem under the rules as provided above.

By way of example, the Pennsylvania Supreme Court has upheld the Disciplinary Board

in imposing severe disciplinary sanctions on a lawyer who violated RPC 1.8 (c) by soliciting a

substantial gift in the will of a client in Office of Disciplinary Counsel v. Jeffrey J. Howell, No.

1635 Disciplinary Docket No. 3 (2008). In that case, the court upheld the suspension of an

attorney for five years when he convinced an elderly client to name him as beneficiary of the

client’s estate and induced his attorney friend to draft the will. Thus, Larry would most likely be

subject to some level of discipline based on the facts of the narrative for violation of RPC 1.8.

6

Page 11: FEBRUARY 2016 PENNSYLVANIA BAR … 2016 PENNSYLVANIA BAR EXAMINATION Essay Questions and Examiners’ Analyses and Performance Test Pennsylvania Board of Law Examiners 601 Commonwealth

4. The final joint1 return of David and Jane for 2015 must report $125,000 of the

insurance payment as income due to an involuntary conversion.

Section 61(a), 26 U.S.C.A. § 61(a) of the Internal Revenue Code (IRC), broadly defines

“gross income” as “all income from whatever source derived,” listing 15 specific, but not all

inclusive, examples. Under section 1033(a) of the IRC, 26 U.S.C.A § 1033(a), a payment from

an insurer for the total loss of a property caused by destruction, theft, seizure, or condemnation,

known as an “involuntary conversion,” must, unless excluded or not recognized by law, be

reported as a gain in the same manner as a voluntary sale to the extent the payment exceeds the

adjusted basis of the property. See id.; 26 CFR 1.1033(a)-2. In the facts above, Jane has decided

not to replace or rebuild the vacation home, accordingly, she must report the gain on the joint tax

return.

In order to determine how much of the $200,000 must be reported as income one would

turn to the other provisions of the IRC. The IRC provides that a gain from the conversion of

property is determined by the following:

(a) Computation of gain or loss. The gain from the sale or other disposition

of property shall be the excess of the amount realized therefrom over the

adjusted basis provided in section 1011 . . . for determining gain . . . .

(b) Amount realized. The amount realized from the sale or other disposition

of property shall be the sum of any money received plus the fair market

value of the property (other than money) received . . . .

(c) Recognition of gain or loss. . . . [T]he entire amount of the gain or loss,

determined under this section, on the sale or exchange of property shall be

recognized.

26 U.S.C. § 1001. In the facts above, David and Jane received only cash in the form of the

insurance check for $200,000. Accordingly, there is no other item to consider in determining the

gain, other than the cash and the adjusted basis.

The basis of a property is defined, in relevant part here, as “the cost of such property.”

26 U.S.C. § 1012. “The adjusted basis for determining a gain or loss . . . shall be the basis

(determined under section 1012) . . . or other applicable sections . . ., adjusted as provided in

section 1016.” 26 U.S.C. § 1011. Section 1016 provides, in relevant part, that “[p]roper

adjustments [to the basis] in respect of the property shall in all cases be made . . . for

expenditures, receipts, losses or other items, properly chargeable to capital account.” 26 U.S.C.

§ 1016.

Under the facts, the cost basis for David and Jane’s destroyed house is its $75,000

structure value. The narrative establishes the purchase price David and Jane paid for the

structure on the property was $75,000. The facts also provide that David and Jane purchased the

1 Jane is permitted to file a joint federal income tax return as a “surviving spouse” because her spouse died during

the taxable year. See 26 CFR 31.3402(l)-1, INTERNAL REVENUE SERV., Publ’n no. 17 (2015).

7

Page 12: FEBRUARY 2016 PENNSYLVANIA BAR … 2016 PENNSYLVANIA BAR EXAMINATION Essay Questions and Examiners’ Analyses and Performance Test Pennsylvania Board of Law Examiners 601 Commonwealth

house for cash, made no additional capital improvement to add to the basis, nor is there any

indication of a mortgage, line of credit or other lien on the property. Thus the adjusted cost basis

for the home would be $75,000. Accordingly, taking the $200,000 check received by the couple

from the insurance company and subtracting the adjusted basis of $75,000, David and Jane

realized a $125,000 gain on the involuntary conversion of their vacation home.

The Internal Revenue Code does provide for an exclusion of up to $500,000 for joint

spousal taxpayers who have used the property as the principal residence for at least two of the

five years preceding the sale or other disposition of the property. See 26 U.S.C. § 121. While

David and Jane had purchased the home ten years prior to its destruction, they had not used the

vacation home as their principal residence for the last seven years. This also means that they had

not used it as their principal residence within at least two of the five years before its destruction.

Accordingly, they are not permitted to exclude the gain on the sale or other conversion of a

primary residence under the IRC. See 26 U.S.C. § 121. The IRC also provides that if within two

years after the end of the tax year in which the gain on the property converted is realized, the

property is rebuilt or replaced in a way that is similar or related in use to the former property, the

income is deferred at the election of the taxpayer to the extent the amount realized by its

conversion exceeds the cost of the new property. 26 U.S.C. § 1033. However, Jane has chosen

to keep the proceeds, as stated in the facts, and chosen not rebuild or replace the destroyed

structure, thus the final joint income tax return filed by Jane and David for 2015 will require that

the $125,000 be reported as income.

8

Page 13: FEBRUARY 2016 PENNSYLVANIA BAR … 2016 PENNSYLVANIA BAR EXAMINATION Essay Questions and Examiners’ Analyses and Performance Test Pennsylvania Board of Law Examiners 601 Commonwealth

Question No. 1: Grading Guidelines

1. Advancement affecting inheritance

Comments: Candidates should recognize that a gift acknowledged in writing as part of an

anticipated inheritance affects the recipient’s share of the donor’s estate.

6 points

2. Distribution of intestate estate

Comments: Candidates should understand the proportionate shares of a surviving spouse and

children of the decedent as provided by Pennsylvania law.

4 points

3. Professional Conduct – lawyer as beneficiary of will

Comments: Candidates should recognize the presumptive prohibition of a non-related lawyer

soliciting or drafting a substantial gift to the lawyer

5 points

4. Federal tax consequences of involuntary conversion

Comments: Candidates should recognize the inclusion in the federal income tax return of the

gain represented by an insurance recovery of a lost structure which was not the primary

residence

5 points

9

Page 14: FEBRUARY 2016 PENNSYLVANIA BAR … 2016 PENNSYLVANIA BAR EXAMINATION Essay Questions and Examiners’ Analyses and Performance Test Pennsylvania Board of Law Examiners 601 Commonwealth

Question No. 2

Amy owns and has operated Fun Fitness since May 2014 from a leased space in a retail plaza

in Z County, Pennsylvania. The space was formerly a hair salon and was not designed to have heavy

items affixed to the interior drywall. The main room is an open space with a floor mat and a wood

bar affixed to a wall. Patrons often use the bar to support their weight as they stretch and exercise

without supervision. Amy designed the facility layout. Ray, an established local contractor,

constructed the improvements, including affixing the wood bar to the wall.

By May 2015, the anchor fixtures supporting the bar had loosened causing the bar to become

unstable and unsafe to use. On May 4, 2015, a patron told Amy of the problem, which was not

obvious on visual inspection. Amy examined the bar and its condition. She immediately contacted

Ray and told him that the bar was unstable and unsafe. Ray immediately went to the business and

inspected the wood bar. He told Amy he would return the following morning, May 5, to repair the

condition and provide additional support for the bar. Amy took no further action. Under Amy’s

lease she is responsible for all construction, maintenance, and repairs to the space.

Carol has been a Fun Fitness member since it opened and uses the facility twice per week.

On May 5, 2015, at 6:15 a.m. Carol arrived for an early workout before Amy or Ray came to the

facility. She proceeded to the exercise room and gripped the wood bar to stretch her hamstring

muscles. This caused the bar to pull away from the wall. Carol lost her balance, fell, and injured her

right shoulder. Carol was treated at a nearby express-care facility and was advised to take non-

prescription pain medication for her shoulder injury.

Amy arrived later that morning and learned about Carol’s accident. Amy called Carol and

told her, “I can’t believe Ray didn’t fix that bar.” Amy told Carol she would pay Carol’s medical

insurance deductible at the express-care facility. Amy then stated to Carol, “Let’s keep this out of

10

Page 15: FEBRUARY 2016 PENNSYLVANIA BAR … 2016 PENNSYLVANIA BAR EXAMINATION Essay Questions and Examiners’ Analyses and Performance Test Pennsylvania Board of Law Examiners 601 Commonwealth

court.” Carol responded that she wanted at least $10,000, which Amy rejected as being ridiculous.

Amy then offered to extend Carol’s membership for two years at no fee, which Carol rejected.

A week after her fall at Fun Fitness, Carol spoke to Linda, who was also a Fun Fitness

member. Carol told Linda she was continuing to experience shoulder pain and stated, “It’s the same

shoulder I hurt when I fell on the ice in my driveway last December and ended up in the emergency

room. I like the doctor treating me now for the Fun Fitness injury more than Dr. Smith who treated

me in December.” In July, Carol had arthroscopic surgery on her right shoulder to repair joint

damage. Carol hired Attorney Able who timely filed a complaint in Z County for her personal

injuries alleging negligence against Amy. Amy later learned from a reputable contractor that the

method Ray used to affix the wood bar to the wall was improper and was completed in an

unworkmanlike manner.

1. What must Carol establish to sustain her cause of action for negligence based on

premises liability against Amy, and with what likelihood of success?

2. Amy feels strongly that she is not responsible for Carol’s injury, or at least not solely

responsible, and that Ray’s incompetence caused the incident. Amy’s Answer is not

yet due. What procedural steps could Amy’s attorney take in the action filed by Carol

against Amy to try to hold Ray legally responsible for the claim asserted by Carol?

3. Assume that this matter proceeds to trial and Carol’s attorney seeks to introduce some

of Amy’s statements to prove that Amy is liable for her injuries. Other than a

possible hearsay objection, on what basis should Amy object, and how should the

court rule on the following statements:

(a) Amy’s statements offering to pay Carol’s medical insurance deductible for her

treatment at the express care facility.

(b) Amy’s offer to extend Carol’s membership at no charge to avoid litigation.

4. Amy’s counsel has properly served Carol with interrogatories asking her to identify

the date and location of any emergency room visits and the names and addresses of

her treating physicians relating to the previous injury from her fall on the ice in

December. If Carol objects on the basis that the interrogatories are overly broad and

not proper and Amy’s counsel moves the court to dismiss the objection, how should

the court rule?

11

Page 16: FEBRUARY 2016 PENNSYLVANIA BAR … 2016 PENNSYLVANIA BAR EXAMINATION Essay Questions and Examiners’ Analyses and Performance Test Pennsylvania Board of Law Examiners 601 Commonwealth

Question No. 2: Examiner’s Analysis

1. Carol is an invitee and will have to prove that Amy knew or reasonably should have

known of the dangerous condition that caused Carol’s injuries, and that Amy failed to

exercise reasonable care to protect or warn patrons against the dangerous condition.

Carol should be successful based on these facts.

The Pennsylvania Superior Court summarized the current law in Pennsylvania regarding

negligence as follows:

Pennsylvania law places the burden on the plaintiff to establish the

existence of negligence on the part of the defendant by proving four

elements: (1) a duty or obligation recognized by law; (2) a breach of

that duty; (3) a causal connection between the conduct and the

resulting injury; and (4) actual damages.

Swift v. Northeastern Hospital of Philadelphia, 456 Pa. Super. 330, 335, 690 A.2d 719, 722 (1997)

appeal denied, 701 A.2d 577 (Pa. 1997), citing Pittsburgh Nat’l Bank v. Perr, 431 Pa.Super. 580,

584, 637 A.2d 334, 336 (1994). The Swift court went on to discuss the duty owed as follows:

The nature of the duty which is owed in any given situation hinges

primarily upon the relationship between the parties at the time of the

plaintiff’s injury. Pittsburgh Nat’l Bank v. Perr, supra. The standard

of care that a possessor of land owes to one who enters upon the land

depends upon whether the entrant is a trespasser, a licensee or an

invitee.

Id. citing Carrender v. Fitterer, 503 Pa. 178, 184, 469, A.2d 120, 123 (1983).

Amy’s duty to Carol on her cause of action in negligence based on premises liability will be

determined by Carol’s status. An "invitee" is defined in the following manner:

(1) An invitee is either a public invitee or a business visitor.

(2) A public invitee is a person who is invited to enter or remain on land as a

member of the public for a purpose for which the land is held open to the

public.

(3) A business visitor is a person who is invited to enter or remain on land for a

purpose directly or indirectly connected with business dealings with the

possessor of the land.

RESTATEMENT (SECOND) OF TORTS § 332 (1965), cited with approval in Atkins v. Urban

Redevelopment Auth. of Pittsburgh, 489 Pa. 344, 351 n. 2, 414 A.2d 100, 103 n. 2 (1980); see also

Kimble v. Mackintosh Hemphill Co., 359 Pa. 461, 470, 59 A.2d 68, 72 (1948) (citing RESTATEMENT

OF TORTS § 332). As a paying member of the exercise facility, Carol is an invitee.

12

Page 17: FEBRUARY 2016 PENNSYLVANIA BAR … 2016 PENNSYLVANIA BAR EXAMINATION Essay Questions and Examiners’ Analyses and Performance Test Pennsylvania Board of Law Examiners 601 Commonwealth

Applying Section 343 of the Restatement (Second) of Torts, the Pennsylvania Superior Court

has explained that,

A party is subject to liability for physical harm caused to an invitee if:

he knows of or reasonably should have known of the condition and the

condition involves an unreasonable risk of harm, he should expect that

the invitees will not realize it or will fail to protect themselves against

it, and the party fails to exercise reasonable care to protect the invitees

against the danger.

Blackman v. Fed. Realty Inv. Tr., 444 Pa.Super. 411, 415, 664 A.2d 139, 142 (1995) citing

RESTATEMENT (SECOND) OF TORTS § 343 (1965). The Superior Court further stated that “[a]n invitee

must prove either the proprietor of the land had a hand in creating the harmful condition, or he had

actual or constructive notice of such condition.” Swift, 456 Pa. Super. at 336; 690 A.2d at 722, citing

Moultrey v. Great Atl. & Pac. Tea Co., 281 Pa.Super. 525, 535, 422 A.2d 593, 598 (1980).

Amy was the business owner and as such she owed a duty of care to her business invitees to

ensure that the premises were free from known dangers and those dangers which might be

“‘discovered with reasonable care.’” See Truax v. Roulhac, 2015 Pa. Super. 217, 126 A.3d 991, 997

(2015) (quoting Charlie v. Erie Ins., Exch., 100 A.3d 244, 253 (Pa. Super. 2014)). Amy knew of the

dangerous condition of the bar as she had been informed by a patron that it was loose, and she

subsequently inspected it. She then contacted Ray, who had originally installed the wooden bar in

an unworkmanlike manner, and who promised to correct the dangerous condition but did not.

Additionally, Amy told Carol, “I can’t believe Ray didn’t fix that bar,” further acknowledging that

she was aware of the dangerous condition. Thus, Amy knew that the bar involved a dangerous

condition and an unreasonable risk of harm. Subsequent to her inspection of the bar, Amy failed to

take any further action such as placing a sign on the bar warning members of its defective condition

and warning them not to use the bar. Amy’s failure to warn of the dangerous condition, or to correct

it prior to Carol’s visit to the facility, is a breach of her duty of care to her invitee, Carol.

The risk that caused Carol’s injury, the unstable condition of the wooden bar, was one that an

invitee could not have been expected to anticipate. There was no warning or notice provided to

those using the room that the bar was unsafe, and the bar’s instability was not obvious on visual

inspection. Carol should be able to show that Amy failed to exercise reasonable care to protect her

invitees against the known danger by failing to repair or to warn of the condition created by the

faulty state of the wooden bar. This failure will likely be deemed to be the proximate and actual

cause of Carol’s injuries, i.e. if Carol had not leaned on the bar, as patrons often did, she would not

have fallen and injured her shoulder. The facts also indicate that Carol suffered actual physical

injury to her shoulder or that her fall aggravated the pre-existing condition for which Amy may be

held liable. The Pennsylvania Supreme Court has held that, a tortfeasor is liable for all injuries

caused by his negligence. “The term ‘injuries’ necessarily refers as much to the aggravating of an

already existing disability as it does to the infliction of an original wound.” Pavorsky v. Engels, 410

Pa. 100, 102-03, 188 A.2d 731, 733 (1963) (further citation omitted). Carol should be successful on

her claim as all of the elements of negligence have been satisfied.

13

Page 18: FEBRUARY 2016 PENNSYLVANIA BAR … 2016 PENNSYLVANIA BAR EXAMINATION Essay Questions and Examiners’ Analyses and Performance Test Pennsylvania Board of Law Examiners 601 Commonwealth

2. Amy’s attorney should file a praecipe for a writ or a complaint against Ray to join him

as an additional defendant.

Carol has filed her personal injury action naming Amy as the defendant on a negligence

theory based on premises liability. Amy as owner of the premises is liable for her negligence to her

invitees. Amy engaged the services of Ray to install the wooden bar and affix it to the wall. Amy

believes that Ray failed to install the bar in a workmanlike manner and that he is liable for Carol’s

injury.

Carol has not named Ray as a defendant. Ray may be liable to Carol for the defective

method of installing the wooden bar. In order to preserve her claim against Ray, Amy should join

him in the action as an additional defendant under Pa.R.C.P. No. 2252.

Rule No. 2252 (a) provides in pertinent part the following:

(a) [A]ny party may join as an additional defendant any person not

a party to the action who may be

(1) solely liable on the underlying cause of action against the

joining party, or

* * * *

(4) liable to or with the joining party on any cause of action

arising out of the transaction or occurrence or series of

transactions or occurrences upon which the underlying

cause of action against the joining party is based.

* * *

(b) The joining party may file as of course a praecipe for a writ or

a complaint.

Pa.R.C.P. No. 2252.

Amy should file either a praecipe for a writ or a complaint to join Ray as an additional

defendant on the basis that Ray should be solely liable on Carol’s cause of action, or liable to, or

with Amy on the cause of action. Amy’s cause of action against Ray for negligent work is based

upon the same occurrence or transaction upon which the plaintiff, Carol, bases her claim; i.e. Carol’s

injuries were caused by Ray defectively installing the bar that pulled away from the wall when Carol

put her weight on it. See Stokes v. Loyal Order of Moose Lodge No. 696, 502 Pa. 460, 466 A.2d

1341 (1983) (holding it was error to permit joinder where complaints did not arise out of the same

transaction or occurrence); 202 Island Car Wash, L.P. v Monridge Construction, Inc., 2006 Pa.Super

362, 913 A.2d 922 (2006) (stating “‘[t]he key inquiry is whether the additional defendant’s liability

is related to the plaintiff’s claim against the original defendant’” (quoting Somers v. Gross, 393 Pa.

Super. 509, 574 A.2d 1056, 1058 (1990)).

As Ray is not a party to the suit, Amy would need to file the above-mentioned praecipe for a

writ or a complaint to join Ray as an additional defendant, no later than “sixty days after the service

14

Page 19: FEBRUARY 2016 PENNSYLVANIA BAR … 2016 PENNSYLVANIA BAR EXAMINATION Essay Questions and Examiners’ Analyses and Performance Test Pennsylvania Board of Law Examiners 601 Commonwealth

upon the original defendant of the initial pleading of the plaintiff or any amendment thereof, or . . .

the time for filing the joining party’s answer . . . whichever is later.” Pa.R.C.P. No. 2253. If the

praecipe for writ or complaint for joinder is not filed within that time period as required by the Rule,

Amy will be required to either get consent of the parties or ask leave of the court. Id.

3. Amy’s attorney should object on the following bases: (a) that an offer to pay medical

expenses is not admissible to show liability, and the court will likely rule it is

inadmissible on that basis; and (b) that an offer of an extended free membership would

be excluded as a compromise offer, and the court will likely also rule it is inadmissible.

Pennsylvania Rules of Evidence provide the test for relevant evidence as follows:

Evidence is relevant if:

(a) it has any tendency to make a fact more or less probable than it

would be without the evidence; and

(b) the fact is of consequence in determining the action.

Pa.R.E. 401. “All relevant evidence is admissible, except as otherwise provided by law.” Pa.R.E.

402.

(a) Carol’s attorney seeks to introduce the offer Amy made to pay Carol’s medical insurance

deductible as relevant evidence to show Amy is liable for Carol’s injuries. Amy’s offer does have a

tendency to make the fact that she was responsible for Carol’s injury more probable, which would be

of consequence in Carol’s negligence cause of action. Thus, Amy’s offer could be admissible if the

court determines that it is relevant to prove liability. However, Amy’s offer, while perhaps relevant,

would not be admissible for the purpose for which it was offered, i.e. to prove Amy’s liability.

Pennsylvania Rule of Evidence 409 provides as follows:

Evidence of furnishing, promising to pay, or offering to pay medical,

hospital, or similar expenses resulting from an injury is not admissible

to prove liability for the injury.

Pa.R.E. No. 409.

Amy’s offer to pay Carol’s express care medical insurance deductible would be an expense

covered by Rule 409 when offered for the purposes of proving Amy’s liability for Carol’s injury. As

such, Amy should not be penalized for extending the offer; the court should sustain the objection,

and rule Amy’s statement offering to pay Carol’s out of pocket medical expenses inadmissible for

the purpose of proving Amy’s liability for Carol’s injury. Pennsylvania’s Rule 409 is identical to the

corresponding federal rule. See Fed. R. Evid. 409; Pa.R.E. 409 cmt. The rationale for the rule is that

“‘such payment or offer is usually made from humane impulses and not from an admission of

liability, and to hold otherwise would tend to discourage assistance to the injured person.’” Fed. R.

15

Page 20: FEBRUARY 2016 PENNSYLVANIA BAR … 2016 PENNSYLVANIA BAR EXAMINATION Essay Questions and Examiners’ Analyses and Performance Test Pennsylvania Board of Law Examiners 601 Commonwealth

Evid. 409 advisory committee note (quoting 20 ALR. 2d 291, superseded by 65 ALR 3d 932); see

also Burns v. Flaherty Co., 278 Pa. 579, 581, 123 A. 486, 496-97 (1924).

(b) Amy’s offer of extended membership at no charge was made to dissuade Carol from

initiating legal action and in response to Carol’s demand of $10,000.

Pennsylvania Rules of Evidence Rule 408 titled Compromise Offers and Negotiations

provides in relevant part as follows:

(a) Prohibited Uses. Evidence of the following is not admissible --on behalf of

any party --either to prove or disprove the validity or amount of a disputed

claim . . . .

(1) furnishing, promising or offering -- or accepting, promising to

accept, or offering to accept -- a valuable consideration in

compromising or attempting to compromise the claim; and

(2) conduct or a statement made during compromise negotiations

about the claim.

Pa.R.E. 408 (a).

In Rochester Machine Corp. v. Mulach Steel Corp., 498 Pa. 545, 549, 449 A.2d 1366, 1368

(1982) the Pennsylvania Supreme Court stated that the threshold inquiry is whether the evidence

between the parties “can be fairly characterized as relating to an offer of compromise.” The court

defined the term “an offer to compromise” as “the settlement of differences by mutual concessions;

an adjustment of conflicting claims.” Rochester, 498 Pa. at 549, 449 A.2d at 1368. In Rochester the

court held that an exchange of letters between the parties, the first demanding repair for damage to a

building and the second letter responding to that demand with the admission that the defendant was

responsible for some of the damage was not part of an offer to compromise and therefore was

admissible. Id. at 555, 449 A.2d at 1371; see also, Hooker v. State Farm Fire and Casualty Co., 880

A.2d 70, 85 (Pa. Cmwlth. 2005) (stating that “[t]aking responsibility for some items of damage

while contesting responsibility for others does not suggest that it is an offer to compromise a

disputed claim”).

In the facts, Amy stated to Carol, “Let’s keep this out of court.” By doing so, Amy was

attempting to begin compromise negotiations. Carol’s response, requesting $10,000 to keep the

matter out of court was part of the negotiations as was Amy’s resulting offer to extend Carol’s

membership for two years at no charge. The membership extension on Amy’s part constitutes

valuable consideration. Thus the two year free membership offer was an offer of compromise under

the Rochester definition of an offer of settlement by mutual concessions. The court, therefore,

should rule that it is inadmissible and excluded under Pa.R.E. 408 (a).

4. Carol’s objections will be unsuccessful and the court will order Carol to answer the

interrogatories.

16

Page 21: FEBRUARY 2016 PENNSYLVANIA BAR … 2016 PENNSYLVANIA BAR EXAMINATION Essay Questions and Examiners’ Analyses and Performance Test Pennsylvania Board of Law Examiners 601 Commonwealth

Discovery under the Pennsylvania Rules of Civil Procedure “is liberally allowed with respect

to any matter, not privileged, which is relevant to the cause being tried.” George v. Schirra, 814

A.2d 202, 204 (Pa. Super. 2002). Rule 4003.1 provides, in pertinent part, the following:

(a) Subject to the provisions of Rules 4003.2 to 4003.5 inclusive and Rule 4011, a

party may obtain discovery regarding any matter, not privileged, which is

relevant to the subject matter involved in the pending action, whether it relates

to the claim or defense of the party seeking discovery or to the claim or

defense of any other party, including the existence, description, nature,

content, custody, condition and location of any books, documents, or other

tangible things and the identity and location of persons having knowledge of

any discoverable matter.

(b) It is not ground for objection that the information sought will be inadmissible

at the trial if the information sought appears reasonably calculated to lead to

the discovery of admissible evidence.

Pa.R.C.P. 4003.1. Under Rule 4005, subject to the limitations set forth in Rule 4011, “a party may

serve upon any other party written interrogatories . . . [which] may relate to any matter” otherwise

discoverable. See Pa.R.C.P. 4003.1.

Although broad in scope, discovery is not permitted which:

(a) is sought in bad faith;

(b) would cause unreasonable annoyance, embarrassment,

oppression, burden or expense to the deponent or any person or

party;

(c) is beyond the scope of discovery as set forth in Rules 4003.1

through 4003.6;

(d) is prohibited by any law barring disclosure of mediation

communications and mediation documents; or

****

(e) would require the making of an unreasonable investigation by

the deponent or any party or witness.

Pa.R.C.P. 4011.

The written interrogatories sent by Amy’s counsel are a proper discovery method under Rule

4005 and were properly served on Carol who is a party to the action. Amy is requesting

identification of Carol’s treating physicians and information relating to the emergency room visit

concerning the prior injury to Carol’s shoulder from her fall on the ice. Answers to these

interrogatories relate to injuries to the same shoulder which Carol reinjured in her fall at Fun Fitness.

Carol’s fall at Fun Fitness is the basis of Carol’s personal injury claim in the current action.

“Evidence is relevant if . . . it has any tendency to make a fact more or less probable than it

would be without the evidence; and . . . the fact is of consequence in determining the action.”

17

Page 22: FEBRUARY 2016 PENNSYLVANIA BAR … 2016 PENNSYLVANIA BAR EXAMINATION Essay Questions and Examiners’ Analyses and Performance Test Pennsylvania Board of Law Examiners 601 Commonwealth

Pa.R.Evid. 401. Thus, the interrogatories are relevant to the subject matter of the pending action and

are reasonably calculated to lead to the discovery of admissible evidence. Identifying the treating

physicians and emergency room information may lead to obtaining relevant information as to the

nature and extent of any prior injury and damage to Carol’s shoulder, which is part of Carol’s current

claim for personal injuries. There is no basis under Pa.R.C.P. 4011 for Carol to object as the

requests are not sought in bad faith. It does not appear that the disclosure of the information would

cause unreasonable annoyance, embarrassment, oppression, burden or expense nor would it require

an unreasonable investigation as the facts show that Carol has the information on who treated her.

Further, the date and location of her emergency room treatment and names of her treating physicians

for the previous injury may lead to relevant evidence regarding the condition of Carol’s shoulder

prior to the current injury which is the basis for her claim in the action against Amy. Under Rule

4006, therefore, the court should dismiss Carol’s objections and order her to answer Amy’s

interrogatories.

18

Page 23: FEBRUARY 2016 PENNSYLVANIA BAR … 2016 PENNSYLVANIA BAR EXAMINATION Essay Questions and Examiners’ Analyses and Performance Test Pennsylvania Board of Law Examiners 601 Commonwealth

Question No. 2: Grading Guidelines

1. Negligence (Premises Liability)

Comments: The candidate should identify the elements of negligence, the duty owed by Amy to

Carol as an invitee of Fun Fitness, and that based on the facts and the breach of that duty Carol will

be successful.

7 Points

2. Joinder of Additional Defendant

Comments: The candidate should recognize that Fun Fitness should protect any claim of joint

liability or sole liability it has against Ray by joining him as an Additional Defendant in the lawsuit

and the options to proceed with joinder under the Pa.R.C.P.

3 Points

3. Evidence - Offer to Pay Medical and Similar Expenses - Compromise Offers and

Negotiations

Comments: The candidate should identify that the offer by Amy to pay Carol’s out-of-pocket

medical expenses and Amy’s offer of an extended membership is a comproise offer and neither will

be admissible to prove liability under the Pa.R.E.

6 Points

4. Discovery- Written Interrogatories to a Party

Comments: The candidate should recognize that the interrogatories are permitted under the

Pa.R.C.P. in that they seek information which is relevant to the subject matter of her action, not

privileged and that the court should overrule Carol’s objections and order her to answer the

interrogatories.

4 Points

19

Page 24: FEBRUARY 2016 PENNSYLVANIA BAR … 2016 PENNSYLVANIA BAR EXAMINATION Essay Questions and Examiners’ Analyses and Performance Test Pennsylvania Board of Law Examiners 601 Commonwealth

Question No. 3

Clyde and Bonnie were married in June of 2010 in C County, Pennsylvania, and continue

to reside in C County. Clyde is currently earning about $100,000 a year as a salesman, and

Bonnie recently lost her job. The couple is very materialistic and often tries to live beyond their

financial means.

Desperate to buy herself a new Mercedes, Bonnie went to a local bank in C County in

early September 2015 and saw a customer, later identified as Jackie, walking towards the front

door of the bank. Bonnie approached Jackie with an unloaded .40 caliber handgun, which

Bonnie was licensed to carry, and pointed the gun at Jackie’s head. Bonnie said, “Give me all

your money.” Jackie, who was terrified at the sight of the gun being pointed at her head,

immediately handed over to Bonnie all of her cash, totaling $500, which Jackie had intended to

deposit in the bank. Bonnie then fled the scene. Jackie was the only witness to the events as no

other persons were in the area at the time of the incident.

Within hours, the police apprehended Bonnie at her home and arrested her for the

incident outside the bank. Before being transported to the police station for booking, Bonnie was

handcuffed and properly informed of her Miranda rights. Bonnie immediately told the police

that she did not wish to speak with them and that she wanted an attorney. While being

transported to the police station, Bonnie, without being asked any questions by the transporting

officer or any other officer, said, “I didn’t mean to scare the woman. I just needed the money to

get a new car.”

A preliminary hearing was held two months after Bonnie’s arrest at which Jackie testified

in detail, under oath, to the facts surrounding the incident cited above. During her testimony,

Jackie unequivocally identified Bonnie as the offender. Jackie was then subjected to a rigorous

20

Page 25: FEBRUARY 2016 PENNSYLVANIA BAR … 2016 PENNSYLVANIA BAR EXAMINATION Essay Questions and Examiners’ Analyses and Performance Test Pennsylvania Board of Law Examiners 601 Commonwealth

cross-examination by Bonnie’s attorney. A stenographer was present for the hearing and later

produced a transcript that he provided to the district attorney and defense counsel. A week after

the preliminary hearing Jackie was tragically killed in a motor vehicle accident.

Shortly after Clyde and Bonnie’s marriage, Clyde bought a home located at 215 Simmons

Street in C County for $100,000, which was well below the fair market value of the property.

While Bonnie was in jail awaiting trial she was served with a Complaint, which was filed in C

County, raising claims for divorce and equitable distribution of property. Shortly thereafter,

Bonnie received reliable information that Clyde was planning to immediately sell the home at

215 Simmons Street to his friend, Mike, and that Clyde would be netting approximately $50,000

from the sale of the home. She further learned that Clyde intended to take the $50,000 and

immediately move to Mexico. This was the only asset of substantial value that Bonnie and

Clyde accumulated during the marriage, and there is no dispute that the $50,000 is considered to

be marital property.

1. Would a robbery charge against Bonnie be supported by the facts presented?

2. If Bonnie’s attorney properly files a pre-trial motion to suppress the statement

Bonnie made to the police officer while being transported to the station on the

basis that it was in violation of her Miranda rights, how should the district

attorney respond, and how would the court likely rule?

3. A police officer was called at Bonnie’s criminal trial to read Jackie’s testimony

from the preliminary hearing, and the defense counsel objects based upon

hearsay. What should be the district attorney’s response, and how should the

court rule?

4. What remedy could be pursued on behalf of Bonnie under the Pennsylvania

Divorce Code to ensure that any proceeds of the sale of the home are not taken by

Clyde and are properly preserved for equitable distribution purposes?

21

Page 26: FEBRUARY 2016 PENNSYLVANIA BAR … 2016 PENNSYLVANIA BAR EXAMINATION Essay Questions and Examiners’ Analyses and Performance Test Pennsylvania Board of Law Examiners 601 Commonwealth

Question No. 3: Examiner’s Analysis

1. A robbery charge against Bonnie is likely supported by the facts presented.

“A person is guilty of robbery if, in the course of committing a theft, [the person] . . .

threatens another with or intentionally puts him in fear of immediate serious bodily injury.” 18

Pa. C.S.A. § 3701(a)(1)(ii). “An act shall be deemed in the course of committing a theft if it

occurs in an attempt to commit theft or in flight after the attempt or commission.” 18 Pa. C.S.A.

§ 3701(a)(2). “Serious bodily injury” is defined as “[b]odily injury which creates a substantial

risk of death or which causes serious, permanent disfigurement, or protracted loss or impairment

of the function of any bodily member or organ.” 18 Pa. C.S.A. § 2301. “A person is guilty of

theft if he unlawfully takes, or exercises unlawful control over, moveable property of another

with the intent to deprive him thereof.” 18 Pa. C.S.A. § 3921(a). In Commonwealth v. Valentine,

101 A.3d 801 (Pa. Super. 2014), the court concluded that a robbery charge was supported under

section 3701 where the defendant appeared from outside the victim’s view while she was alone

at a public transit bus stop, pointed a gun a few inches from her face, threatened to shoot her, and

demanded she hand over her purse and cellular telephone. The victim testified that she was

afraid, shocked, and nervous and did not know whether the defendant would shoot her if she said

the wrong thing. Id. at 807. A robbery charge may also be sustained where, in the course of

committing a theft, a person takes or removes property from another person by force, however

slight. 18 Pa. C.S.A. § 3701(a)(1)(v). “Constructive force consists of the use of threatening

words or gestures sufficient to separate the victim from his property.” McElrath v.

Commonwealth, 592 A.2d 740, 745 (Pa. Super. 1991).

As applied here, the facts indicate that Bonnie went to the local bank in C County and

approached Jackie as she was walking towards the front door of the bank. As Bonnie

approached Jackie, Bonnie pointed her .40 caliber handgun at Jackie’s head and stated, “Give me

all of your money.” “The Commonwealth need not prove a verbal utterance or threat to sustain a

conviction under subsection 3701(a)(1)(ii). It is sufficient if the evidence demonstrates

aggressive actions that threatened the victim’s safety.” Commonwealth v. Alford, 880 A.2d 666,

676 (Pa. Super. 2005) (quoting Commonwealth v. Hopkins, 747 A.2d 910, 914-15 (Pa. Super.

2000)). Bonnie’s actions terrified Jackie and caused her to hand over the $500 in cash which she

had in her possession. These facts clearly demonstrate that Bonnie threatened Jackie with

immediate serious bodily injury, namely shooting her in the head. The facts also support

Bonnie’s intent to place Jackie in fear of such injury. The fact that the gun was unloaded will

have no bearing on the ultimate result because Jackie did not know that the gun was unloaded. It

is also clear that Bonnie’s threats towards Jackie were made in the course of committing the theft

of $500 from Jackie as the threats were made as she was in the process of taking the money from

Jackie, and it can be inferred that it was Bonnie’s intent to permanently deprive Jackie of the

money.

Alternatively, it can be argued that a robbery charge is supported under Section 3701

(a)(1)(v) as constructive force was used in order to separate Jackie from her property. By

threatening Jackie with her gun Bonnie forced Jackie to turn over her money and this was

accomplished through the use of force.

22

Page 27: FEBRUARY 2016 PENNSYLVANIA BAR … 2016 PENNSYLVANIA BAR EXAMINATION Essay Questions and Examiners’ Analyses and Performance Test Pennsylvania Board of Law Examiners 601 Commonwealth

The facts likely support a robbery charge against Bonnie relative to the incident with

Jackie.

2. The district attorney should respond that although Bonnie was in custody and had

invoked her Miranda rights, the statement she made was unsolicited and not in

response to any interrogation. The court would likely rule that Bonnie’s statement

is admissible.

In order to secure the constitutional right against self-incrimination, the United States

Supreme Court has held that confessions and other statements obtained through custodial

interrogation are inadmissible as evidence unless the declarant is warned before questioning, in

clear and unequivocal terms, that: (1) he or she has the right to remain silent; (2) that any

statement he or she makes may be used as evidence against the declarant; (3) that he or she has

the right to consult with an attorney and to have an attorney present during interrogation; and (4)

that if the declarant is indigent, an attorney will be appointed to represent him or her. Miranda v.

Arizona, 384 U.S. 436, 479 (1966). Pa. Const., Art. 1, Section 9, has language very similar to

the Fifth Amendment and provides that no person can be compelled to give evidence against

himself. Commonwealth v. Molina, 104 A.3d 430, 442 (Pa. 2014).; Commonwealth v. Arroyo,

723 A.2d 162, 166 (Pa. 1999). The test for determining whether or not a person is in custody

for Miranda purposes is whether the person is physically deprived of her “freedom of action in

any significant way or is placed in a situation in which [s]he reasonably believes that [her]

freedom of action o[r] movement is restricted by such interrogation.” Commonwealth v. O’Shea,

318 A.2d 713, 715 (Pa. 1974); see also Miranda, 384 U.S. at 444. An interrogation occurs, and

Miranda warnings are required, when the person is in custody and “where the police should

know that their words or actions are reasonably likely to elicit an incriminating response from

the suspect.” Commonwealth v. Gwynn, 723 A.2d 143, 149 (Pa. 1998); Rhode Island v. Innis,

446 U.S. 291 (1980). Once a person invokes his or her right to counsel all questioning must stop

until counsel is obtained or provided unless the accused initiates further communication,

exchanges or conversations with the police. Edwards v. Arizona, 451 U.S. 477, 484-85 (1981);

Commonwealth v. Zook, 553 A.2d 920 (Pa. 1989). Where a defendant who has invoked his or

her right to counsel offers an “unsolicited” or “gratuitous” statement, the statement will not be

subject to suppression. Commonwealth v. Hughes, 639 A.2d 763, 771 (Pa. 1994).

As applied here, the facts indicate that Bonnie was placed under arrest at her home and

was subsequently handcuffed and placed in the police cruiser to be transported to the police

station for booking. When she was placed under arrest, the facts provide that she was properly

administered her Miranda warnings. Upon receiving her warnings, she immediately informed

the police that she did not wish to speak with them and wanted an attorney. Since Bonnie was

clearly in custody at this point the police would be prohibited from interrogating her with regard

to the incident in question. The facts also specify that as Bonnie was being transported to the

police station, and without being asked any questions by the transporting or any other officer,

Bonnie said, “I didn’t mean to scare the woman. I just needed the money to get a new car.”

Since this statement was made voluntarily by Bonnie and not in response to any police

questioning, it will likely be deemed to be an “unsolicited” or “gratuitous” statement, and the

statement will not be subject to suppression.

23

Page 28: FEBRUARY 2016 PENNSYLVANIA BAR … 2016 PENNSYLVANIA BAR EXAMINATION Essay Questions and Examiners’ Analyses and Performance Test Pennsylvania Board of Law Examiners 601 Commonwealth

In sum, despite the fact that Bonnie was in custody and had invoked her Miranda rights

at the time she gave the statement in question, it is likely that it will be admitted into evidence as

it was voluntarily given and it was not in response to any interrogation or questioning by the

police.

3. The district attorney should respond that although this evidence would be deemed

to be hearsay, it should be admitted under the former testimony exception to the

hearsay rule as Jackie will now be deemed to be unavailable for trial. The court will

likely admit the testimony.

Hearsay is defined as “a statement that . . . the declarant does not make while testifying at

the current trial or hearing; and [that] a party offers in evidence to prove the truth of the matter

asserted in the statement.” Pa.R.E. 801. A statement includes “a person’s oral assertion . . . if

the person intended it as an assertion.” Id. A declarant “means the person who made the

statement.” Id. “Hearsay is not admissible except as provided by the [Pennsylvania Rules of

Evidence], by other rules prescribed by the Pennsylvania Supreme Court, or by statute.” Pa.R.E.

802. It is clear that Jackie made oral assertions at the preliminary hearing which would be

deemed to be statements under the hearsay rule. Since Jackie, who would be deemed to be the

declarant of these statements, would not be making the statements while testifying at trial and

this evidence is being offered at trial to prove the truth of the matter asserted in the statements,

the statements would be deemed to be hearsay. Thus, the district attorney would have to identify

an exception to the hearsay rule in order to have Jackie’s statements admitted at trial.

Pennsylvania Rule of Evidence 804 provides for exceptions to the Hearsay Rule when the

declarant is deemed to be unavailable as a witness. A declarant is considered to be unavailable

as a witness if the declarant cannot testify at trial because of death. Pa.R.E. 804(a)(4).

Pennsylvania Rule of Evidence 804(b)(1) provides for an exception to the hearsay rule where the

declarant is unavailable as a witness and has provided former testimony. In order for this

exception to apply, the former testimony must have been given by the declarant as a witness at a

proceeding enumerated in the rule, including hearings, and be offered against a party who had an

opportunity and similar motive to develop the testimony by cross examination. Pa.R.E.

804(b)(1). The use of prior recorded testimony where a witness is unavailable and where the

defendant had counsel and an opportunity to fully cross-examine the witness at that prior

proceeding, is not a violation of the Sixth Amendment right of confrontation. Commonwealth v.

Bazemore, 614 A.2d 684, 685 (Pa. 1992)

The facts provide that Jackie was the victim of the offense committed outside the bank

and that she was the only witness (other than Bonnie) to the event. The facts also indicate that

Jackie testified in detail to the facts surrounding the incident, which are outlined above, and that

she identified Bonnie as the offender. The facts show that this testimony was given at a

preliminary hearing, under oath, and that Jackie was subjected to a rigorous cross examination by

Bonnie’s attorney. A stenographic transcript of the preliminary hearing was later provided to

counsel for both the defense and the prosecution. Unfortunately, Jackie is now deceased and will

not be available to testify at trial. Since Jackie is now deceased, and she was the only witness to

24

Page 29: FEBRUARY 2016 PENNSYLVANIA BAR … 2016 PENNSYLVANIA BAR EXAMINATION Essay Questions and Examiners’ Analyses and Performance Test Pennsylvania Board of Law Examiners 601 Commonwealth

the events, the district attorney will have to introduce Jackie’s preliminary hearing testimony at

trial in order to prove the charges against Bonnie.

Although the testimony from the preliminary hearing would be deemed to be hearsay, it

is likely that the district attorney would be able to introduce this evidence at trial due to the fact

that Jackie would be deemed to be unavailable, and her testimony would likely be admissible

under the former testimony exception to the hearsay rule.

4. Special relief should be sought on behalf of Bonnie pursuant to the Pennsylvania

Divorce Code in order to prevent the dissipation of proceeds received from the sale

of the marital home.

Special relief under the Pennsylvania Divorce Code is governed by 23 Pa. C.S.A. §

3323(f) as follows:

In all matrimonial causes, the court shall have full equity power and jurisdiction

and may issue injunctions or other orders which are necessary to protect the

interests of the parties or to effectuate the purposes of this part and may grant

such other relief or remedy as equity and justice require against either party or

against any third person over whom the court has jurisdiction and who is involved

in or concerned with the disposition of the cause.

Further, where it appears to the court that a party is about to remove property of that party

from the jurisdiction of the court or dispose of property in order to defeat equitable distribution,

an injunction may be issued to prevent the removal or disposition, and the property may be

attached as prescribed by general rules. 23 Pa. C.S.A. § 3505(a). In Lazovitz v. Lazovitz, 453

A.2d 615, 619-620 (Pa. Super. 1982) the court found that the entry of a preliminary injunction

was appropriate where the trial court restrained the husband from, inter alia, removing marital

assets from Pennsylvania to another state.

As applied here, Bonnie has learned that Clyde is taking steps to sell the home at 215

Simmons Street which is expected to net $50,000 in cash. She has further learned that it is

Clyde’s intention to take the $50,000, which is deemed to be marital property, and immediately

move to Mexico. In order to prevent the removal and possible dissipation of this marital asset,

Bonnie’s counsel should immediately petition the court for special relief to protect against her

husband’s attempts to defeat her rights to a monetary share of the marital home. The court

should be asked to have the funds placed in escrow, or some similar arrangement, until such time

as an agreement can be reached on the distribution of the asset. In short, the equitable powers of

the Court should be immediately invoked to prevent the dissipation of this marital asset.

25

Page 30: FEBRUARY 2016 PENNSYLVANIA BAR … 2016 PENNSYLVANIA BAR EXAMINATION Essay Questions and Examiners’ Analyses and Performance Test Pennsylvania Board of Law Examiners 601 Commonwealth

Question No. 3: Grading Guidelines

1. Criminal Law

Comments: The Candidate should discuss the elements of robbery, apply the applicable facts,

and conclude that a robbery charge is likely supported on these facts.

5 Points

2. Criminal Procedure

Comments: The candidate should discuss the applicable rules regarding Miranda, apply the

applicable facts, and conclude that the Court will likely admit Bonnie’s statement.

5 Points

3. Evidence

Comments: The candidate should discuss the hearsay rule and former testimony exception,

apply the applicable facts and conclude that the Court will likely permit Jackie’s testimony from

the Preliminary Hearing to be presented at trial.

7 Points

4. Family Law

Comments: The candidate should recognize that special relief should be sought on behalf of

Bonnie pursuant to the Pennsylvania Divorce Code to ensure that the net proceeds of the sale of

the home are properly preserved for equitable distribution purposes.

3 Points

26

Page 31: FEBRUARY 2016 PENNSYLVANIA BAR … 2016 PENNSYLVANIA BAR EXAMINATION Essay Questions and Examiners’ Analyses and Performance Test Pennsylvania Board of Law Examiners 601 Commonwealth

Question No. 4

Gloria resides in Village in State Y. Gloria wants to sell her car. She has placed an 8 by

11 inch For Sale sign in the window of her car that she parks on the street in front of her home.

A Village police officer saw the sign and has cited Gloria under Section 17 of the Village Code

(the “Code”), which provides: “It shall be unlawful to park any vehicle displaying a sign

advertising the vehicle for sale on any street in Village. This section shall not apply to the

advertising of a vehicle for sale while driving the vehicle.” The violation carries a $100 fine.

Currently, Gloria’s neighbor has a 2 by 3 foot sign in her front yard stating her child is an

“A” student. Another neighbor has a “Mary Kate Cosmetics For Sale” sign in her car window

that she parks in the street. These signs are permissible under the Code.

Gloria has filed suit to preclude enforcement of Section 17 of the Code alleging it

violates her First Amendment commercial speech rights under the U. S. Constitution. Village

had enacted Section 17 based on its mayor’s opinion that For Sale signs in parked cars are

dangerous distractions and that the Village would look like a used car lot if they were not

banned. Meeting minutes at the time of the vote on the ordinance evidence that a well-known,

credible, national traffic safety study, which determined that For Sale signs in parked vehicles

did not increase accidents among pedestrians or motorists, was presented and disregarded.

Gloria owns GP, Inc. (“GP”), a State Y corporation. GP is a printing business with 150

employees. Its sole facilities, operations, and customers are in State Y. Last year, GP advertised

to hire a graphic designer for a position that required a bachelor’s degree in graphic design.

Amber was interviewed, stated she had the required bachelor’s degree, and was hired.

Amber recently had a child and has just returned to work. Ted, Amber’s supervisor, told

her that she would have difficulty balancing work and childcare responsibilities, and discharged

her. Ted replaced her with a male graphic designer who has an infant child. Amber has filed a

27

Page 32: FEBRUARY 2016 PENNSYLVANIA BAR … 2016 PENNSYLVANIA BAR EXAMINATION Essay Questions and Examiners’ Analyses and Performance Test Pennsylvania Board of Law Examiners 601 Commonwealth

discrimination suit against GP under Title VII of the Civil Rights Act in State Y federal district

court alleging disparate treatment due to her sex. She is seeking reinstatement, back pay, and

front pay. During discovery, GP learned that Amber does not have a bachelor’s degree in

graphic design. If known at the time, GP would have terminated Amber.

Heather, a State X resident, worked for GP. When she was hired she signed a valid non-

compete/non-disclosure agreement (the “Agreement”) that prohibited disclosure of confidential

GP information and barred her from working for GP competitors in State Y or any adjoining

state for one year after termination of her employment. The Agreement was signed at GP’s

offices.

Heather recently quit and took a job with New Co., a GP competitor located in adjoining

State X. Using GP customer information (confidential under the Agreement) that she had taken

from GP employee Sally’s confidential files, Heather began soliciting GP customers for New Co.

GP has sued Heather in State Y’s eastern federal district court for breach of the Agreement,

seeking over $100,000 in damages. The State Y court is known for its expertise in handling non-

compete/non-disclosure lawsuits and for rendering prompt decisions.

1. At the non-jury trial on Gloria’s First Amendment claim, Village took the position

that Section 17 is a valid regulation of commercial speech. Based on the facts set

forth above, how should the court analyze the merits of Gloria’s commercial

speech claim and with what outcome?

2. Assume Amber has established a prima facie case of discrimination against GP

and GP offers evidence of her lack of the required degree as the reason to support

her termination. (a) What effect, if any, would this evidence have on GP’s ability

to defend against or defeat the discrimination claim? And (b) What effect, if any,

would it have on the remedies that Amber claims?

3. Heather has filed a Motion in State Y to transfer venue to the congested western

federal district court in State X, 80 miles from the State Y federal eastern district,

on the grounds that she works and lives in State X. How would the court analyze

the Motion to Transfer, and with what result?

28

Page 33: FEBRUARY 2016 PENNSYLVANIA BAR … 2016 PENNSYLVANIA BAR EXAMINATION Essay Questions and Examiners’ Analyses and Performance Test Pennsylvania Board of Law Examiners 601 Commonwealth

Question No. 4: Examiner’s Analysis

1. The court should analyze Gloria’s First Amendment claim using the Supreme Court’s

four-part test announced in Central Hudson to determine if Village’s restriction on

commercial speech is constitutional. The court will likely find that Village’s restriction

of commercial speech prohibiting For Sale signs in vehicles parked on public streets is

unconstitutional.

The First Amendment provides, in relevant part, that “Congress shall make no law . . .

abridging the freedom of speech.” U.S. Const. amend. I. This prohibition is made applicable to the

States by the Fourteenth Amendment. Central Hudson Gas & Elec. Corp. v. Public Serv. Comm’n,

447 U.S. 557, 561 (1980) (citation omitted). Gloria’s First Amendment claim challenging Section

17 of the Village Code stems from her right to free speech, in particular, commercial speech. The

Supreme Court has defined commercial speech as “expression related solely to the economic

interests of the speaker and its audience.” Id. (citation omitted). Concerning commercial speech, the

Supreme Court stated:

The commercial marketplace, like other spheres of our social and cultural life,

provides a forum where ideas and information flourish. Some of the ideas and

information are vital, some of slight worth. But the general rule is that the speaker

and the audience, not the government, assess the value of the information presented.

Thus, even a communication that does no more than propose a commercial

transaction is entitled to the coverage of the First Amendment.

Edenfield v. Fane, 507 U.S. 761, 767 (1993).

Historically, the protection afforded commercial speech was not as great as that given other

constitutionally guaranteed forms of expression. Central Hudson, 447 U.S. at 563 (citing Ohralik v.

Ohio State Bar Ass'n, 436 U.S. 447, 455–56 (1978)). In Central Hudson, the Court instructed that

the “protection available for particular commercial expression turns on the nature both of the

expression and of the governmental interests served by its regulation.” Id. at 563. To that end, the

Court held that “there can be no constitutional objection to the suppression of commercial messages

that do not accurately inform the public about lawful activity[,] [and] [t]he government may ban

forms of communication more likely to deceive the public than to inform it.” Id. (citation omitted).

Where a targeted “communication is neither misleading nor related to unlawful activity,” the Court

directed that the government “must assert a substantial interest to be achieved by restrictions on

commercial speech. Moreover, the regulatory technique must be in proportion to that interest.” Id. at

564.

To effectuate its review, the Supreme Court adopted a four-part test to determine whether the

regulation before it violated rights in commercial speech:

(1) whether the speech at issue concerns lawful activity and is not misleading;

(2) whether the asserted government interest is substantial; and, if so,

(3) whether the regulation directly advances the governmental interest

asserted; and

29

Page 34: FEBRUARY 2016 PENNSYLVANIA BAR … 2016 PENNSYLVANIA BAR EXAMINATION Essay Questions and Examiners’ Analyses and Performance Test Pennsylvania Board of Law Examiners 601 Commonwealth

(4) whether it is not more extensive than is necessary to serve that interest.

Central Hudson, 447 U.S. at 566. The Court labeled this an “intermediate standard of review.”

Edenfield, 507 U.S. at 767. The Supreme Court has also noted that “[t]he four parts of the Central

Hudson test are not entirely discrete. All are important and, to a certain extent, interrelated: Each

raises a relevant question that may not be dispositive to the First Amendment inquiry, but the answer

to which may inform a judgment concerning the other three.” Greater New Orleans Broad. Ass'n,

Inc. v. United States, 527 U.S. 173, 183-84 (1999).

Applying the Central Hudson factors to the issue here, it appears that on its face, the Village

Code restricts commercial speech by prohibiting advertising a vehicle for sale using a For Sale sign

in any vehicle parked on the street. The Village would have the burden of justifying its prohibition

under Section 17 of the Code to Gloria’s sign. See Edenfield, 507 U.S. at 770 (citation omitted)

(stating that “‘[t]he party seeking to uphold a restriction on commercial speech carries the burden of

justifying it’” (quoting Bolger v. Youngs Drug Products Corp., 463 U.S. 60, 71, n. 20 (1983)).

Applying the Central Hudson test to the facts here, Gloria’s For Sale sign advertising the sale of her

car neither pertains to unlawful activity nor is there any suggestion that it is misleading.

Accordingly, the first prong of the Central Hudson test is satisfied.

Second, the asserted government interests suggested by the Code are to improve aesthetics

and traffic safety. The Supreme Court has held that a government’s interest in preserving aesthetics

can be substantial. See, e.g., Members of City Council v. Taxpayers for Vincent, 466 U.S. 789, 805

(1984) (providing “the state may legitimately exercise its police powers to advance aesthetic

values”). A majority of Justices have likewise recognized that traffic safety is a “substantial

governmental goal[].” Metromedia, Inc. v. City of San Diego, 453 U.S. 490, 507-08, 528 (1981)

(plurality), (Justices Brennan, with whom Justice Blackmun joined, concurring in the judgment and

stating they had “no quarrel with the substantiality of the city's interest in traffic safety.” 453 U.S. at

528.) Strictly speaking then, the interests in traffic safety and aesthetics asserted by the mayor in

support of the ordinance are likely substantial government interests for purposes of the Central

Hudson test.

It is unlikely, however, that a court applying the third prong of Central Hudson’s test would

find an immediate connection between the ban of For Sale signs in parked cars and the

government’s aim of improving aesthetics and traffic safety. Section 17 does not prohibit other

types of signs that would be equally distracting to drivers and pedestrians. For example, Gloria’s

neighbors both had signs, one of which was larger than hers and another that also advertised

products for sale that did not violate the Code. The facts also provide that the Code was passed

based on the mayor’s personal belief that For Sale signs in parked cars are dangerous distractions

and that permitting them would make the Village resemble a used car lot. However, when the Code

was enacted Village disregarded a well-known and credible study that found that For Sale signs in

parked vehicles did not increase accidents among pedestrians or motorists. Additionally, the

Village’s concern for aesthetics rings hollow given that the Code allows larger signs along Village

streets and other signs in cars other than For Sale signs advertising that vehicle.

There is not an obvious “immediate connection” between the government’s goal of improved

aesthetics and traffic safety and the regulation banning For Sale signs in cars, but instead only a

weak connection. Moreover, that connection appears to be solely based upon the mayor’s

30

Page 35: FEBRUARY 2016 PENNSYLVANIA BAR … 2016 PENNSYLVANIA BAR EXAMINATION Essay Questions and Examiners’ Analyses and Performance Test Pennsylvania Board of Law Examiners 601 Commonwealth

speculation. Therefore, it is unlikely that the Village could sustain its burden as to this prong. See

Edenfield, 507 U.S. at 770-71 (explaining that the government’s “burden is not satisfied by mere

speculation or conjecture; rather, a governmental body seeking to sustain a restriction on commercial

speech must demonstrate that the harms it recites are real and that its restriction will in fact alleviate

them to a material degree”). Consequently, the court would likely conclude that Section 17 of the

Code does not directly advance the claimed substantial interests of the municipality in safety and

aesthetics. See Pagan v. Fruchey, 492 F.3d 766 (6th Cir. 2007) (holding affidavit of town’s police

chief containing conclusory statement that interest in road safety and aesthetics was the reason for

the ban was insufficient to show how the ban on parking of vehicles with potentially distracting

signs inside them would advance those goals in a direct and material way).

Finally, even if the court found the third prong was met, it would still have to address the

fourth prong; i.e., whether the challenged restriction is more extensive than is necessary to serve the

stated interest. The fourth prong of the test does not require government to use the least restrictive

means to achieve its goals, but it does demand “a reasonable fit between the legislature's ends and

the means chosen to accomplish those ends, a means narrowly tailored to achieve the desired

objective.” Lorillard Tobacco Co. v. Reilly, 533 U.S. 525, 556 (2001). The Supreme Court has

stated, that “if there are numerous and obvious less-burdensome alternatives to the restriction on

commercial speech, that is certainly a relevant consideration in determining whether the ‘fit’

between ends and means is reasonable.” City of Cincinnati v. Discovery Network, Inc., 507 U.S.

410, 418, n. 12 (1993).

The Village Code is likely not sufficiently narrow to justify the regulation because, while the

Village proffered important goals, the law in question was not tailored to achieve its stated goals. To

achieve its goal, it is easy to imagine a less-burdensome alternative to banning For Sale signs in

parked cars. Village could impose parking regulations prohibiting parking cars on busy streets,

limiting the time when the vehicle can be parked, or the number of such cars that display a For Sale

sign; any of which would be more narrowly tailored to advance the goal of traffic safety. Moreover,

other signs are permitted that are more distracting and perhaps less aesthetically pleasing than a

small For Sale sign in a car; for example, signs advertising jewelry or cosmetics for sale that are not

prohibited. Thus, it does not appear that there is a reasonable fit between the Village’s goals of

aesthetics and traffic safety and the prohibition of For Sale signs in parked cars. Accordingly, the

Village has likely not satisfied the fourth prong of the Central Hudson test. See Burkow v. City of

Los Angeles, 119 F. Supp. 2d 1076 (C.D. Cal. 2000) (holding that Los Angeles’ ban on For Sale

signs in the windows of parked cars on public streets was unconstitutional).

The ordinance described in the question would likely be found to be an unconstitutional

violation of Gloria’s commercial speech rights. Gloria would, therefore, likely prevail in precluding

enforcement of Section 17 of the Code.

2. (a) Amber’s misrepresentation that she had a bachelor’s degree when GP hired her

will not have any effect on GPs ability to defend against Amber’s discrimination claim

when GP discharged her due to her sex.

Title VII of the Civil Rights Act of 1964, 42 U.S.C.A. § 2000e et seq., as amended, prohibits

discrimination in employment on the basis of sex. Specifically, an employer may not “fail or refuse

to hire or . . . discharge any individual, or otherwise . . . discriminate against any individual with

31

Page 36: FEBRUARY 2016 PENNSYLVANIA BAR … 2016 PENNSYLVANIA BAR EXAMINATION Essay Questions and Examiners’ Analyses and Performance Test Pennsylvania Board of Law Examiners 601 Commonwealth

respect to his compensation, terms, conditions, or privileges of employment, because of such

individual's . . . sex.” 42 U.S.C.A. § 2000e-2(a)(1). Amber was terminated based on a sex

stereotype concerning her ability to juggle motherhood and her job, which constitutes discrimination

under Title VII. See e.g., Phillips v. Martin Marietta Corp., 400 U.S. 542 (1971) (holding that Title

VII's prohibition against sex discrimination precludes employers, absent a bona fide occupational

qualification, from having policies that discriminate on the basis of sex plus other factors such as

having pre-school age children.) Here Amber’s prima facia case of sex discrimination is assumed

for purposes of this question.

Under the facts above, GP did not learn of Amber’s misrepresentation until after she was

terminated for discriminatory reasons, i.e. that she had a child. GP will not be able to use evidence

of Amber’s misrepresentation of her qualifications as a legitimate non-discriminatory reason for

termination because GP did not learn of the misrepresentation prior to discharging her for

discriminatory reasons. In McKennon v. Nashville Banner Publishing Company, 513 U.S. 352

(1995), the Supreme Court held that an employee was not automatically barred from all recovery

under the Age Discrimination in Employment Act (ADEA) when the employer discharged her for

discriminatory reasons and later discovered misconduct by that employee that would have resulted in

her discharge for nondiscriminatory reasons. Thus the Court recognized that an employer could not

replace the discriminatory motive in discharging an employee with a nondiscriminatory one when

the evidence shows the employer did not know of the misconduct prior to the discharge. Id. at 360.

Although McKennon arose in the context of the ADEA, its holding that after-acquired evidence does

not provide a basis to defeat liability for a claim of discrimination is applicable to Title VII claims.

See McKennon, 513 U.S. at 358 (stating that “[t]he ADEA and Title VII share common substantive

features and also a common purpose: ‘the elimination of discrimination in the workplace’”) (citation

omitted).

The McKennon holding has also been applied in Title VII cases where after-acquired

evidence was discovered pertaining to the employee's misrepresentations in a job application rather

than misconduct during employment. See Mardell v. Harleysville Life Ins. Co., 65 F.3d 1072 (3d

Cir. 1995) cert. granted, vacated and remanded 514 U.S. 1034 (instructing Third Circuit to apply

McKennon’s holding, i.e., after discovered evidence of misconduct is relevant to damages but does

not preclude liability, in unlawful discrimination context); Wehr v. Ryan's Family Steak Houses,

Inc., 49 F.3d 1150, 1153 (6th Cir.1995) (examining resume fraud; “we are persuaded by

[McKennon's] language that it applies equally to a Title VII claim.”) aff’d after remand, 99 F.3d

1140 (1996); Wallace v. Dunn Construction Company, 62 F.3d 374 (lst Cir. 1995) (en banc)

(holding McKennon applied to Title VII case where after-discharge acquired evidence showed that

an employee made a misrepresentation on job application). Accordingly, Amber’s misrepresentation

that she had a bachelor’s degree in graphic design will not have any effect on GP’s ability to defend

against her discrimination claim.

2. (b) Amber’s misrepresentation that she had a bachelor’s degree when GP hired her

will likely affect her ability to receive front pay and reinstatement for her

discriminatory discharge.

After-acquired evidence of misconduct does influence the remedy available to an employee

terminated for discriminatory reasons. As a general rule, neither reinstatement nor front pay is an

appropriate remedy in such circumstances. McKennon, 513 U.S. at 361-62; Wallace, 62 F.3d at 380

32

Page 37: FEBRUARY 2016 PENNSYLVANIA BAR … 2016 PENNSYLVANIA BAR EXAMINATION Essay Questions and Examiners’ Analyses and Performance Test Pennsylvania Board of Law Examiners 601 Commonwealth

(citing McKennon). “Where an employer seeks to rely upon after-acquired evidence of wrongdoing,

it must first establish that the wrongdoing was of such severity that the employee in fact would have

been terminated on those grounds alone if the employer had known of it at the time of the

discharge.” McKennon, 513 U.S. at 362-63. Where, as here, the after-acquired evidence was of

such severity that it would have led to termination of the employee had the employer known about it,

back pay generally will be limited to the period from the date of the unlawful discharge to the date

the evidence of misconduct was discovered. McKennon, 513 U.S. at 362; Wallace, 62 F.3d at 380

(citing McKennon). The rationale for the rule is that the employer’s exercise of legitimate

prerogatives should not be obstructed and the employee should not be placed in a better position than

she would have occupied absent the discrimination. “Cutting off relief at the time that a legitimate

discharge would have occurred accomplishes these ends.” Shattuck v. Kinetic Concepts, Inc., 49

F.3d 110, 1108-09 (6th Cir. 1995) (interpreting McKennon).

The facts establish that a bachelor’s degree in graphic design was a requirement for Amber’s

position, and that she would have been terminated if the misrepresentation of her qualifications had

been discovered. Although Amber will prevail in her claim of discriminatory termination despite

her misrepresentation, she will likely not be entitled to reinstatement or front pay if GP credibly

testifies that it would have discharged her had it discovered that she did not have a bachelor’s degree

in graphic design. Accordingly, Amber will only be eligible for an award of back pay from the date

of the unlawful discharge to the date the new information was discovered. See McKennon, 513 U.S.

at 362.

3. In ruling on the Motion to Transfer Venue, it is likely that a court would conclude

venue in State X is not more convenient, nor would transfer promote the interests of

justice such as to warrant transfer from the State Y court to State X because State Y is

the location where the material and significant events that underlie the breach action

occurred. Accordingly, Heather’s Motion to Transfer Venue to State X would likely

not succeed.

Generally, in the absence of a specific statutory provision, proper venue is determined by

applying the following:

A civil action may be brought in—

(1) a judicial district in which any defendant resides, if all defendants are residents

of the State in which the district is located;

(2) a judicial district in which a substantial part of the events or omissions giving

rise to the claim occurred, or a substantial part of property that is the subject of the

action is situated; or

(3) if there is no district in which an action may otherwise be brought as provided

in this section, any judicial district in which any defendant is subject to the court's

personal jurisdiction with respect to such action.

33

Page 38: FEBRUARY 2016 PENNSYLVANIA BAR … 2016 PENNSYLVANIA BAR EXAMINATION Essay Questions and Examiners’ Analyses and Performance Test Pennsylvania Board of Law Examiners 601 Commonwealth

28 U.S.C.A. § 1391 (b). Even where original venue is proper, the federal rules allow for a party to

seek transfer of venue to a more convenient forum. The statute entitled Change of Venue, 28

U.S.C.A. § 1404(a), provides the following:

(a) For the convenience of parties and witnesses, in the interest of justice, a district

court may transfer any civil action to any other district or division where it might

have been brought or to any district or division to which all parties have consented.

“A motion to transfer venue should be granted upon a showing that the transferee venue is

‘clearly more convenient’ than the venue chosen by the plaintiff.” In re Genentech, Inc., 566 F.3d

1338, 1342 (Fed. Cir. 2009) (citation omitted).

In this case, the parties have not consented to any particular district, and the facts support the

conclusion that the case could have been brought in either State X or State Y. Substantial events

giving rise to GP’s lawsuit for breach of the non-disclosure/non-compete Agreement (Agreement)

have occurred in both states. Heather’s employment agreement was executed in State Y, but her

employment with New Co., that perpetuates her breach of the Agreement, is located in State X.

Therefore, the federal district court would weigh whether transfer will be granted for the convenience

of the parties and witnesses and in the interest of justice. 28 U.S.C.A. § 1404(a).

“In the typical case not involving a forum-selection clause, a district court considering a §

1404(a) motion (or a forum non conveniens motion) must evaluate both the convenience of the

parties and various public-interest considerations. Ordinarily, the district court would weigh the

relevant factors and decide whether, on balance, a transfer would serve ‘the convenience of parties

and witnesses’ and otherwise promote ‘the interest of justice.’” Atlantic Marine, 134 S. Ct. at 581

(citing 28 U.S.C. § 1404(a) (footnote omitted)). The Atlantic Marine Court noted:

Factors relating to the parties' private interests include “relative ease of access to

sources of proof; availability of compulsory process for attendance of unwilling, and

the cost of obtaining attendance of willing, witnesses; possibility of view of premises,

if view would be appropriate to the action; and all other practical problems that make

trial of a case easy, expeditious and inexpensive.” Piper Aircraft Co. v. Reyno, 454

U.S. 235, 241, n. 6 . . . (1981) Public-interest factors may include “the administrative

difficulties flowing from court congestion; the local interest in having localized

controversies decided at home; [and] the interest in having the trial of a diversity case

in a forum that is at home with the law.” Ibid. . . . The Court must also give some

weight to the plaintiffs' choice of forum.

Id. at 581 n. 6 (citing Norwood v. Kirkpatrick, 349 U.S. 29 (1955) (stating that the relevant factors

considered under section 1404(a) are the same as those considered under the doctrine of forum non

conveniens)). The district court should “evaluate both the convenience of the parties and various

public-interest considerations” guided by the factors set forth in Atlantic Marine for a venue transfer

analysis. See Norwood, 349 U.S. at 32. "Section 1404(a) is intended to place discretion in the

district court to adjudicate motions for transfer according to an `individualized, case-by-case

consideration of convenience and fairness.’" Stewart Org., Inc. v. Ricoh Corp., 487 U.S. 22, 29

(1988) (quoting Van Dusen v. Barrack, 376 U.S. 612, 622 (1964).

34

Page 39: FEBRUARY 2016 PENNSYLVANIA BAR … 2016 PENNSYLVANIA BAR EXAMINATION Essay Questions and Examiners’ Analyses and Performance Test Pennsylvania Board of Law Examiners 601 Commonwealth

Although the Supreme Court did not mandate consideration of any specific factors in

evaluating a motion to transfer venue, federal courts interpreting Rule 1404(a) have recognized the

following:

(1) the convenience of the witnesses; (2) the location of relevant documents and

the relative ease of access to sources of proof; (3) the convenience of the parties;

(4) the locus of operative facts; (5) the availability of process to compel the

attendance of unwilling witnesses; (6) the relative means of the parties; (7) a

forum's familiarity with the governing law; (8) the weight accorded a plaintiff's

choice of forum; and (9) trial efficiency and the interests of justice, based on the

totality of the circumstances.

Kelling v. Hartford Life & Acc. Ins. Co., 961 F. Supp. 2d 1216, 1218 (M.D. Fla. 2013) (quoting

Manuel v. Convergys Corp., 430 F.3d 1132, 1135 n.1 (11th Cir. 2005)); Jumara v. State Farm Ins.

Co., 55 F.3d 873, 879-80 (3d Cir. 1995) (referring to 15 CHARLES ALAN WRIGHT ET AL. FEDERAL

PRACTICE & PROCEDURE § 3848 AT 385 (2D ED. 1986), and 1A Pt. 2 MOOORE’S FEDERAL PRACTICE ¶

0.345[5]. The Jumara court outlined similar factors stating that there is “no definitive formula or list

of factors[,]” but that courts have considered the following:

(1) the plaintiff’s choice of forum; (2) the defendant’s preference; (3) where the claim

arose; (4) the convenience of the parties; (5) the convenience of the witnesses, but

only to the extent that the witnesses may actually be unavailable for trial in one of the

fora; (6) the location of books and records, similarly limited to the extent that the files

could not be produced in the alternative forum; (7) the enforceability of the judgment;

(8) practical considerations that could make the trial easy, expeditious, or

inexpensive; (9) the relative court congestion in the competing courts; (10) the local

interest in deciding local controversies at home; (11) the public policies of the fora;

(12) and the familiarity of the trial judge with the applicable state law in diversity

cases.

Id.

Examining several of the above and the Atlantic Marine factors, it is likely the Motion to

Transfer Venue to State X would be denied because State X is not a more convenient forum than

State Y based on the considerations underlying Rule 1404.

Concerning the parties’ private interests:

Convenience of Witnesses/Parties

According to the facts, Gloria and Heather will likely be witnesses that will testify at a trial.

Heather lives and works in State X and Gloria lives and works in State Y. GP customers

solicited by Heather are located in State Y and will likely be called to testify about Heather’s

solicitation. Sally, a GP employee located in State Y, had the confidential list of GP

customers and will likely be called to testify to same. Thus, this factor weighs in favor of

venue in State Y.

35

Page 40: FEBRUARY 2016 PENNSYLVANIA BAR … 2016 PENNSYLVANIA BAR EXAMINATION Essay Questions and Examiners’ Analyses and Performance Test Pennsylvania Board of Law Examiners 601 Commonwealth

Location of relevant documents and the relative ease of access to sources of proof

The sources of proof mentioned in the question are the non-disclosure/non-compete contract

executed in State Y and the list of GP customers located in State Y. Additional

documentation would include the information about any customers Heather did or did not

solicit on behalf of New Co. and would likely be in State X. Accordingly, this factor appears

to slightly favor venue in State Y.

Location of operative facts

The operative facts are that Heather executed a valid non-compete/non-disclosure Agreement

in State Y. She was performing her duties under the Agreement in State Y. She breached the

Agreement, first, by taking the job with New Co. in adjoining State X and, second, by taking

the names of GP customers. She then began soliciting GP customers on behalf of New Co.,

also in breach of her agreement, while working for New Co. in State X. The operative facts

are situated predominantly in State Y, with some arguably in State X. This factor is

relatively neutral, but still favors venue in State Y.

The plaintiff’s choice of forum

GP’s suit was filed in the eastern district of State Y. This factor favors State Y.

Practical considerations that could make the trial easy, quick, or inexpensive

The question specifies that the distance between the plaintiff’s and defendants’ chosen forum

is 80 miles. This distance would not likely be seen as prohibitive. However, most witnesses

are located in State Y. Thus, this factor favors state Y.

As to the public interests involved:

Administrative difficulties flowing from court congestion

The facts specify that Heather moved for transfer to the congested State X federal district

court. Because State X is congested, this factor would weigh against State X as the preferred

venue.

The forum's familiarity with the governing law

In the question, it is specified that the federal district court in State Y has expertise in non-

compete lawsuits and, therefore, generally renders decisions promptly, which would weigh in

favor of venue in State Y.

Overall, it appears the defendant would not sustain her burden to show that State X was a

more convenient forum considering the relevant factors under Rule 1404(a). The events that

occurred in State Y are not only numerous, they are material and directly give rise to GP’s breach

claim. Pursuant to section 1391(b)(2) of the federal venue statute, a civil action may be brought in

“a judicial district in which a substantial part of the events or omissions giving rise to the claim

occurred, or a substantial part of property that is the subject of the action is situated[.]” 28 U.S.C. §

1391(b)(2). “[I]n determining whether events or omissions are sufficiently ‘substantial’ to support

venue, a court should ’not focus only on those matters that are in dispute or that directly led to the

filing of the action,’ but rather ‘should review ’the entire sequence of events underlying the claim.’”

Prod. Grp. Int'l, Inc. v. Goldman, 337 F. Supp. 2d 788, 798 (E.D. Va. 2004) (citations omitted).

Here, focusing on the entire sequence of events underlying the breach of the non-compete agreement

claim, the events would likely be seen as supporting venue in State Y. Moreover, GP, as the

36

Page 41: FEBRUARY 2016 PENNSYLVANIA BAR … 2016 PENNSYLVANIA BAR EXAMINATION Essay Questions and Examiners’ Analyses and Performance Test Pennsylvania Board of Law Examiners 601 Commonwealth

plaintiff, is entitled to some deference in its choice of forum. There is “a strong presumption in

favor of the plaintiff's choice of forum, which may be overcome only when the private and public

factors clearly point toward trial in the alternative forum.” Prod. Grp. Int'l, 337 F. Supp. 2d at 799

citing Piper Aircraft Co. v. Reyno, 454 U.S. 235, 255 (1981). Consistent with relevant authority, it

is likely that the Motion for Transfer to State X Heather filed will be denied.

37

Page 42: FEBRUARY 2016 PENNSYLVANIA BAR … 2016 PENNSYLVANIA BAR EXAMINATION Essay Questions and Examiners’ Analyses and Performance Test Pennsylvania Board of Law Examiners 601 Commonwealth

Question No. 4: Grading Guidelines

1. Constitutional Law – Commercial Speech

Comments: Applicants should demonstrate an understanding of the 4 part Central Hudson test and

how it balances the government’s interests in regulation with the restriction on speech, and apply the

facts to those principles to reach a well reasoned conclusion.

8 points

2. Employment Law - After Discovered Evidence of Misrepresentation of Qualifications

Comments: Applicants should recognize that although a misrepresentation related to a job

requirement could serve as grounds for termination, this fact cannot serve as a legitimate reason for

termination where an employer was not aware of it at that time. Applicants should also recognize

that the misrepresentation of qualifications for the position may have an impact on the remedies

available for discrimination, notably that Amber will not be eligible for reinstatement or front pay.

5 points

3. Civil Procedure – Venue

Comments: Applicants should recognize the parameters for determining venue in a civil case.

Applicants should apply these standards to the facts in the question to determine where venue is

most convenient to reach a well-reasoned conclusion.

7 points

38

Page 43: FEBRUARY 2016 PENNSYLVANIA BAR … 2016 PENNSYLVANIA BAR EXAMINATION Essay Questions and Examiners’ Analyses and Performance Test Pennsylvania Board of Law Examiners 601 Commonwealth

Question No. 5

In June 2010, John, a wealthy philanthropist, validly conveyed Blackacre, an old

industrial building in Big City, Pennsylvania, in fee “to the Big City School District (School

District) and its successors and assigns so long as Blackacre is used as a trade school.” John died

in 2011. By valid will, John left any interest in real estate that he owned at his death to Alice.

In January 2013, Fred and Wilma, a married couple, purchased “Homestead,” a tall

building located directly across from Maulers’ Stadium, the home of Big City’s professional

football team. On the top floor of the building were two luxury residences – Suite A, which had

an unobstructed and close-up view of the entire interior of the stadium, and Suite B. The deed

conveying Homestead granted title to Fred and Wilma as “husband and wife.” Fred and Wilma

lived in Suite A. On March 15, 2015, they leased Suite B to Alice for a term of two years.

On January 2, 2015, Peach, Inc., a computer, telephone, and media conglomerate,

announced that it was sponsoring a continuous two day rock concert at Maulers’ Stadium on

June 26 - 27, 2015, to launch its new subscription music streaming service. Peach promoted the

concert as a “once-in-a-lifetime” event featuring performances by the greatest assemblage of

rock stars since Woodstock. The concert sold out within minutes of tickets being put on sale.

Since they planned to be out of town at the time of the concert, Fred and Wilma

advertised the availability of Suite A to view the concert on an online classified list. Ralph saw

the ad and called Fred to express his interest in renting Suite A to view the concert. During the

inspection, Fred told Ralph, “The view from here is so good that you almost feel as if you are

inside the Stadium. I gave up my Mauler season tickets because of this view. This will be a

great place for you to watch that concert.” Ralph agreed and signed an agreement to rent Suite A

from Fred and Wilma during the concert dates for $10,000 with payment due on June 25, 2015.

39

Page 44: FEBRUARY 2016 PENNSYLVANIA BAR … 2016 PENNSYLVANIA BAR EXAMINATION Essay Questions and Examiners’ Analyses and Performance Test Pennsylvania Board of Law Examiners 601 Commonwealth

On April 15, 2015, one month after Alice moved into Homestead, the only elevator

servicing Suite B began breaking down so often that it was practically inoperable. When Alice

complained to Fred, he responded, “You can stand to lose a few pounds. Walking up the 15

flights of stairs will be good for you.” On June 9, 2015, a pump used to fill a tank to supply

water to Suite B failed. As a result, Alice had no water for drinking, bathing, and flushing her

toilets. Alice complained numerous times in person, by telephone, and by e-mails to Fred and

Wilma about the lack of water and the elevator. Despite Wilma’s insistence that Fred act on

Alice’s complaints, no repairs were made. Finally, after several weeks, Alice notified Fred and

Wilma in writing, “If you think that ignoring my complaints is going to force me out of my

home, forget it. I’m staying! But I’m not paying a penny in rent until you fix the elevator and

the water.”

1. (a) What property interests were created by John’s conveyance of Blackacre to

the School District?

(b) On July 1, 2015, the School District closed the trade school that it had

operated at Blackacre since the transfer in 2010 due to declining enrollment.

What effect, if any, does the School District’s action have on who owns

Blackacre?

2. On June 23, 2015, all of the rock stars scheduled to perform at the concert

unexpectedly announced that they would not appear because Peach refused to pay

union-scale royalties on their music that Peach made available on its subscription

music streaming service. As a result of this completely unforeseen event, Peach

immediately and permanently cancelled the concert. When Ralph refused to pay

the $10,000 for the rental of Suite A citing the concert’s cancellation, Fred and

Wilma sued Ralph for breach of contract. What contract law theory should Ralph

use in defending this suit, and will it be successful?

3. Fred and Wilma sued Alice for unpaid rent and possession of Suite B. Would

Alice be successful in defending this suit if she asserted the following defenses:

(a) constructive eviction; and (b) breach of an implied warranty of habitability?

4. Due to Fred’s failure to act on Alice’s complaints about Suite B, Fred and Wilma

separated and subsequently divorced. What was the nature of the ownership of

Homestead prior to and after the divorce?

40

Page 45: FEBRUARY 2016 PENNSYLVANIA BAR … 2016 PENNSYLVANIA BAR EXAMINATION Essay Questions and Examiners’ Analyses and Performance Test Pennsylvania Board of Law Examiners 601 Commonwealth

Question No. 5: Examiner’s Analysis

1. (a) John’s conveyance of Blackacre to the School District granted a present fee

simple determinable interest in Blackacre to the School District. John retained a

possibility of reverter.

A fee simple determinable is an interest in land which has the potential to last forever like

a fee simple but is subject to a limitation that causes the grantee’s estate to end and revert to the

grantor upon the occurrence of an event specified in the limitation. BOYER, RALPH E.,

HOVENKAMP, HERBERT, & KURTZ, SHELDON F., THE LAW OF PROPERTY: AN INTRODUCTORY

SURVEY, § 5.2, at p. 83 (West Publishing Co. 4th ed. 1991). The use of words of limitation in a

deed such as “so long as,” “during,” “while,” and “until” is commonly viewed as manifesting an

intent to create a fee simple determinable in the grantee. Brown v. Haight, 435 Pa. 12, 17, 255

A.2d 508, 511 (1969); Higbee Corp. v. Kennedy, 286 Pa. Super. 101, 107, 428 A.2d 592, 595

(1981). John used the words “so long as” in his deed conveying Blackacre. By his use of these

particular words of limitation, John granted a fee simple determinable interest in Blackacre to

School District.

Because the grantee’s interest can end upon the happening of a specified event whenever

a fee simple determinable is created, the grantor is always deemed to have retained a future

contingent interest in the property. BOYER, supra, § 7.3, at p. 162. The future interest retained by

the grantor whenever there is the grant of a fee simple determinable is called a possibility of

reverter. Emrick v. Bethlehem Township, 506 Pa. 372, 378, 485 A.2d 736, 739 (1984). By

granting a fee simple determinable to the School District, John also retained a possibility of

reverter.

1. (b) Upon the occurrence of the events stated in the limitation, the School District’s

fee simple determinable interest in Blackacre automatically ended. Because John

had transferred his possibility of reverter in his will, Alice now owns Blackacre in

fee simple absolute.

Unlike a fee simple subject to a condition subsequent, which requires an action on the

part of the grantor to retake or perfect title upon the happening of the event stated in the

condition, a fee simple determinable interest in land automatically ends upon the occurrence of

the event specified in the limitation, and title is transferred to the holder of the possibility of

reverter. Emrick, 506 Pa. at 379, 485 A.2d at 739. When Blackacre was no longer used as a

trade school as specified in John’s deed, School District’s fee simple determinable interest

ended. At that point, John would have become the owner of the property in fee simple absolute

but for his death.

Pennsylvania law has long recognized that a possibility of reverter is “capable of

transmission by inheritance, conveyance or release.” Herr v. Herr, 957 A.2d 1280, 1285 (Pa.

Super. 2008), (quoting, London v. Kingsley, 368 Pa. 109, 116, 81 A.2d 870, 873 (1951)). The

facts state that John transferred any interest in real estate that he owned at his death by will to

Alice. This would have included the possibility of reverter that John retained in Blackacre.

41

Page 46: FEBRUARY 2016 PENNSYLVANIA BAR … 2016 PENNSYLVANIA BAR EXAMINATION Essay Questions and Examiners’ Analyses and Performance Test Pennsylvania Board of Law Examiners 601 Commonwealth

Alice need take no action. The title to Blackacre automatically vested in Alice, the

holder of the possibility of reverter, when Blackacre was no longer used as a trade school.

2. Ralph should be able to successfully defend the suit brought by Fred and Wilma

based upon the doctrine of frustration of purpose.

In defense to the suit brought by Fred and Wilma for the non-payment of the $10,000

under the agreement to rent Suite A, Ralph should raise the doctrine of frustration of purpose.

The doctrine of supervening frustration or frustration of purpose is described in the Restatement

(Second) of Contracts as follows:

Where, after a contract is made, a party’s principal purpose is substantially

frustrated without his fault by the occurrence of an event the non-occurrence of

which was a basic assumption on which the contract was made, his remaining

duties to render performance are discharged, unless the language or the

circumstances indicate the contrary.

RESTATEMENT (SECOND) OF CONTRACTS, § 265 (Am. Law Inst. 1981). Pennsylvania law has

long recognized the doctrine of frustration of purpose as a valid defense to the performance of a

contract. Alvino v. Carraccio, 400 Pa. 477, 482, 162 A.2d 358, 361 (1960); Greek Catholic

Congregation of Olyphant Borough v. Plummer, 338 Pa. 373, 382, 12 A.2d 435, 439 (1940).

The doctrine of frustration of purpose specifically applies to a situation where some

change in circumstance has so destroyed the value of a party’s performance as to frustrate the

other party’s purpose in making the contract. To establish a defense under this doctrine, four

conditions must be met: (1) “the purpose that is frustrated must have been a principal purpose of

that party in making the contract” and the other party was aware of that purpose; (2) “the

frustration must be substantial[;]” (3) “the non-occurrence of the frustrating event must have

been a basic assumption on which the contract was made[;]” and (4) the frustration must occur

without the fault of the party claiming frustration. RESTATEMENT (SECOND) OF CONTRACTS, §

265, cmts. a & b; see also, JOHN E. MURRAY, JR., MURRAY ON CONTRACTS, § 115 (B), at 734-35

(5th ed. 2011). In considering the doctrine of frustration, courts are directed to examine the

circumstances in which the contract was made in order to determine if the bargain was made on

the basis that a “particular thing or state of things” would continue to exist. Alvino, 400 Pa. at

482, 162 A.2d at 361.

The facts in this case, which are substantially similar to Krell v. Henry, 2 KB 740 (1903),

the famous “Coronation Case,” support the conclusion that Ralph’s duty to pay Fred and Wilma

for the rental of Suite A should be excused based upon frustration of the contract’s purpose.

Ralph’s principal purpose in renting Suite A clearly was to view the concert. The facts

specifically state that after seeing the ad, Ralph called Fred to express his interest in renting Suite

A to view the concert. Further, when Ralph inspected Suite A, Fred told him, “This will be a

great place for you to watch that concert.” Thus, both parties not only were aware that the

concert was a basic assumption on which the contract was made, but that it also was Ralph’s

principal purpose in agreeing to rent Suite A.

42

Page 47: FEBRUARY 2016 PENNSYLVANIA BAR … 2016 PENNSYLVANIA BAR EXAMINATION Essay Questions and Examiners’ Analyses and Performance Test Pennsylvania Board of Law Examiners 601 Commonwealth

The Restatement provides that a party’s frustration of purpose is considered “substantial”

when it is “so severe that it is not fairly to be regarded as within the risks . . . assumed under the

contract.” RESTATEMENT (SECOND) OF CONTRACTS, § 265, cmt. a. Here, there is no indication

that the parties gave any thought to a possibility that the concert would not go off as scheduled or

that they intended Ralph to bear the risk of loss for that possibility. Moreover, the facts

described the supervening event that caused the frustration of purpose as being “completely

unforeseen.” The foreseeability of the supervening event is a factor that must be considered in

determining whether the risk of loss should be assumed by the party seeking to be excused from

the contract. MURRAY ON CONTRACTS, supra, at 735. Based upon the severity of the frustration,

the absence of any allocation of risk, and the unforeseen nature of the supervening event, Ralph’s

frustration of purpose would be considered “substantial” as Ralph’s purpose in renting the suite

was to watch the now-cancelled concert.

Finally, the supervening event that caused the frustration of purpose did not happen

through any fault on the part of Ralph. The facts state that the supervening event that caused the

frustration of purpose was Peach’s refusal to pay the concert’s performers union scale royalties

on the performers’ music made available on Peach’s subscription streaming service.

Once a party has demonstrated a frustration of purpose, that party’s obligation to render

performance is discharged unless the language or the circumstances indicate the contrary.

RESTATEMENT (SECOND) OF CONTRACTS, § 265. “Once … frustration of purpose occurs, ‘it is up

to the parties to waive the difficulties or seek to terminate the agreement.’” Hart v. Arnold, 884

A.2d 316, 335 (Pa. Super. 2005) (quoting, Ellwood City Forge Corp. v. Fort Worth Heat

Treating Co., Inc., 431 Pa. Super. 240, 249, 636 A.2d 219, 223 (1994)). Because Ralph can

demonstrate the elements necessary for asserting the defense of frustration of purpose, his duty

to pay Fred and Wilma for the rental of Suite A was discharged. Accordingly, Ralph’s use of the

doctrine of frustration of purpose in defense of the suit brought by Fred and Wilma for non-

payment of the rent for Suite A will be successful.

3. (a) Alice’s defense to the suit of Fred and Wilma seeking unpaid rent and

possession of Suite B based upon constructive eviction would not be successful.

Under Pennsylvania law, there is an implied covenant of quiet enjoyment of the premises

in every lease. Under this implied covenant, a landlord promises that neither the landlord nor

anyone with a paramount title will wrongfully interfere with the tenant’s enjoyment and

possession of the leased premises. BOYER, supra, § 9.7, at 273-74.

The wrongful actions of the landlord which interfere with the tenant’s enjoyment and

possession of the leased premises, in whole or in part, are considered an eviction. Kelly v.

Miller, 249 Pa. 314, 316-17, 94 A. 1055, 1056 (1915). Forcible or physical expulsion of the

tenant from the leased premises is not necessary to constitute an eviction. Adler v. Sklaroff, 154

Pa. Super. 444, 447, 36 A.2d 231, 233 (1944). The modern view is that an eviction can

constructively occur when a landlord “deprives a tenant of the beneficial enjoyment of the

demised premises and manifests an intent to hold adversely to the tenant.” Walnut-Juniper Co.

v. McKee, Berger & Mansueto, Inc., 236 Pa. Super. 1, 5, 344 A.2d 549, 551 (1975).

43

Page 48: FEBRUARY 2016 PENNSYLVANIA BAR … 2016 PENNSYLVANIA BAR EXAMINATION Essay Questions and Examiners’ Analyses and Performance Test Pennsylvania Board of Law Examiners 601 Commonwealth

A tenant’s allegations of an eviction, if proven, suspend the obligation to pay rent and

constitute a valid defense to liability under a lease. Id. To properly assert the defense of

constructive eviction, the tenant must prove two things. First, the tenant must show that the

interference by the landlord with the tenant’s enjoyment of the leased premises is of a substantial

nature and so injurious to the tenant that it deprives him of the beneficial enjoyment of all or part

of the leased premises. Second, the tenant must in fact give up or abandon possession of the

premises. Kuriger v. Cramer, 345 Pa. Super. 595, 609, 498 A.2d 1331, 1338 (1985).

In this case, the failure to repair the elevator and the pump providing water to Suite B for

an extended period most likely would be regarded as substantially interfering with Alice’s

beneficial enjoyment of all or part of Suite B. The facts, however, state that Alice continued to

reside in Suite B. Because she did not abandon the leased premises, Alice would not be

successful in asserting a defense of constructive eviction to the suit of Fred and Wilma seeking

unpaid rent and possession of Suite B.

3. (b) Alice’s defense to the suit based upon breach of the implied warranty of

habitability would be successful.

Pennsylvania law also implies a warranty of habitability in all residential leases. Under

this warranty, the landlord represents that the leased premises will be free of defects “of a nature

and kind which will prevent the use of the dwelling for its intended purpose to provide premises

fit for habitation by its dwellers.” Pugh v. Holmes, 486 Pa. 272, 289, 405 A.2d 897, 905 (1979).

A breach of the implied warranty of habitability may serve not only as the basis of a complaint,

but also as a defense or a counterclaim to a landlord’s suit for rent or possession. Kuriger, 345

Pa. Super. at 606, 498 A.2d at 1336.

To assert a claim of breach of the implied warranty of habitability, a tenant first must

show that the defect in the leased premises is material. Materiality of the breach is a question to

be decided by the trier of fact on a case-by-case basis and is to be determined by examination of

factors such as regulatory code standards and the nature, seriousness, and duration of the defect.

Pugh, 486 Pa. at 289, 405 A.2d at 905-06. “‘Additionally, . . . a tenant must prove [that] he or

she gave notice to the landlord of the defect or condition, that [the landlord] had a reasonable

opportunity to make the necessary repairs, and the that [the landlord] failed to do so.’” Staley v.

Bouril, 553 Pa. 112, 117, 718 A.2d 283, 285 (1998) (quoting, Pugh, 486 Pa. at 290, 405 A.2d at

906).

A number of remedies are available to a tenant who can demonstrate a breach of the

implied warranty of habitability. These remedies include: (1) the termination of the obligation

to pay rent where the tenant surrenders the possession of the premises; (2) full or partial rent

abatement where the tenant remains in possession of the premises; (3) the right for the tenant to

repair the defect or condition and to deduct the cost of the repair from the rent; and (4) other

traditional contract remedies such as specific performance. Pugh, 486 Pa. at 291-95, 405 A.2d at

907-08.

The facts here support Alice’s successful assertion of a breach of the implied warranty of

habitability as a defense to Fred and Wilma’s suit. Alice can argue to the trier of fact that the

44

Page 49: FEBRUARY 2016 PENNSYLVANIA BAR … 2016 PENNSYLVANIA BAR EXAMINATION Essay Questions and Examiners’ Analyses and Performance Test Pennsylvania Board of Law Examiners 601 Commonwealth

lack of an operating elevator in a tall building and the lack of water for drinking, bathing, and

toileting are material violations of health and safety standards in modern housing and health

codes. Additionally, Alice gave Fred and Wilma notice of the defects on the leased premises and

an opportunity to make repairs by making numerous telephone calls and sending e-mails for

several weeks complaining about the conditions at Suite B. Finally, Alice exercised a remedy

expressly recognized for breach of the implied warranty of habitability: withholding rent. Based

on the total material breach of the implied warranty of habitability, Alice’s obligation to pay rent

was abated in full, and the action for possession would fail because there would not be any

unpaid rent.

Based upon the stated facts, Alice would be successful in asserting the breach of the

implied warranty of habitability as a defense to Fred and Wilma’s suit seeking unpaid rent and

possession of Suite B.

4. Fred and Wilma received title to Homestead as tenants by the entireties. Upon their

divorce, the tenancy by the entireties was severed and each would own an undivided

one-half interest in Homestead as tenants in common.

Tenancy by the entireties is a form of joint ownership of property that exists only

between a husband and a wife. Maxwell v. Saylor, 359 Pa. 94, 96, 58 A.2d 355, 356 (1948).

Under Pennsylvania law, the conveyance of real estate to two grantees who are husband and wife

is presumed to create a tenancy by the entireties unless the deed shows a different intent. Holmes

Estate, 414 Pa. 403, 406, 200 A.2d 745, 747 (1964). The deed to Homestead stated that title to

the property was conveyed to Fred and Wilma as “husband and wife.” Based upon this

language, Fred and Wilma initially owned Homestead as tenants by the entireties.

A tenancy by the entireties is predicated upon legal unity of husband and wife as one

person or unit. Beihl v. Martin, 236 Pa. 519, 522, 84 A. 953, 954 (1912). From the inception of

the estate, “each spouse is seized of the whole or the entirety and not of a share, moiety, or

divisible part.” Gasner v. Pierce, 286 Pa. 529, 531, 134 A. 494, 495 (1926) (citation omitted).

Because a tenancy by the entireties is grounded in the conception of the estate as a single

indivisible unit, neither spouse can terminate or sever the tenancy by his own conveyance or take

any action to adversely affect the other’s right of survivorship and rights to enjoy and possess the

property. 1 RONALD M. FRIEDMAN, LADNER PENNSYLVANIA REAL ESTATE LAW, § 8.04(c)

(George T. Bisel Co., Inc. 6th ed. 2013) (citations omitted).

The legal unity of person existing in a tenancy by the entireties, however, is destroyed by

the divorce of the parties. The general rule set forth in Section 3507 of the Pennsylvania Divorce

Code is that a divorce severs the tenancy by the entireties and the resulting joint interest of the

parties in the property becomes a tenancy in common. 23 Pa. C.S. § 3507(a).

A tenancy in common is an estate in which there is unity of possession but separate and

distinct titles. In Re Estate of Quick, 588 Pa. 485, 490, 905 A.2d 471, 474 (2006). A right of

survivorship does not exist in a tenancy in common. FRIEDMAN, supra, § 8.03.

45

Page 50: FEBRUARY 2016 PENNSYLVANIA BAR … 2016 PENNSYLVANIA BAR EXAMINATION Essay Questions and Examiners’ Analyses and Performance Test Pennsylvania Board of Law Examiners 601 Commonwealth

Under the facts, title to Homestead was conveyed to Fred and Wilma as tenants by the

entireties. Their subsequent divorce severed the tenancy by the entireties and created a tenancy

in common. As tenants in common, Fred and Wilma each own an undivided one-half interest in

the property.

46

Page 51: FEBRUARY 2016 PENNSYLVANIA BAR … 2016 PENNSYLVANIA BAR EXAMINATION Essay Questions and Examiners’ Analyses and Performance Test Pennsylvania Board of Law Examiners 601 Commonwealth

Question No. 5: Grading Guidelines

1. Fee Simple Determinable and Possibility of Reverter

Comments: Candidates should correctly identify the applicable interests as a fee simple

determinable and a possibility of reverter. Candidates should discuss what language is needed to

create a fee simple determinable and the rights associated with a possibility of reverter.

5 Points

2. Frustration of Purpose

Comments: Candidates should recognize the applicability of the defense of supervening

frustration or frustration of purpose. Candidates should discuss the elements necessary to assert

a defense of frustration of purpose and apply these elements to the stated facts in the course of

reaching a well-reasoned conclusion.

6 Points

3. Constructive Eviction and Implied Warranty of Habitability in a Residential Lease

Comments: Candidates should discuss the elements necessary to assert constructive eviction and

breach of the implied warranty of habitability as a defense to an action and the remedies

available to a party asserting these defenses. Candidates should analyze the stated facts in

reaching a well-reasoned conclusion regarding whether these two theories can be successfully

asserted as defenses.

6 Points

4. Severance of a Tenancy by the Entireties by Divorce

Comments: Candidates should recognize that a tenancy by the entireties is a form of joint

ownership of property that exists only between a husband and a wife and discuss Pennsylvania

law’s presumption that a conveyance to a husband and wife creates a tenancy by the entireties.

Candidates should recognize that a divorce severs the tenancy by the entireties and results in the

joint interest of the parties in the property becoming a tenancy in common with each co-tenant

owning an undivided one-half interest in the property.

3 Points

47

Page 52: FEBRUARY 2016 PENNSYLVANIA BAR … 2016 PENNSYLVANIA BAR EXAMINATION Essay Questions and Examiners’ Analyses and Performance Test Pennsylvania Board of Law Examiners 601 Commonwealth

Question No. 6

Farm, Inc. (“Farm”) is a Pennsylvania corporation that operates several fruit farms in

Pennsylvania. Farm has its fruit processed into various jams and jellies. Farm does not own a

commercial kitchen for processing its fruit. Instead, Farm pays Kitchen, Inc. (“Kitchen”) to

process and package its fruit for sale by Farm in jars labeled “Farm Jams and Jellies.”

Farm has two shareholders, Bob and Karen, who serve as Farm’s board of directors and

officers. When Farm was formed, Bob provided the cash needed for it to begin operations and

received 90% of Farm’s stock. Karen, an experienced fruit grower, agreed to operate Farm with

the understanding that Bob would transfer stock to her over time until they became 50/50

owners. Bob has refused numerous requests from Karen to transfer stock to her.

Karen has learned that Kitchen is for sale. If Farm acquired Kitchen, which it has the

financial capability to do, Farm could increase its profit margins and streamline fruit processing.

Frustrated by Bob’s unwillingness to transfer stock, Karen has decided not to tell Bob about

Kitchen being for sale and, instead, has decided to secretly buy Kitchen on her own and keep the

processing profits for herself.

Three weeks ago Farm purchased a used tractor from Equipment, a Pennsylvania

equipment retailer. Karen was told the tractor was used. Equipment made no representations

regarding the quality or suitability of the tractor for Farm’s needs. The sales agreement, signed

by Karen, stated in bold print on page one: “THIS TRACTOR IS SOLD AS IS AND WITH

ALL FAULTS.” Since its purchase, Farm has only been able to use the tractor two days. It

otherwise has repeatedly been in the shop for various repairs.

Blackacre is a 100 acre tract in Pennsylvania that adjoins one of Farm’s parcels. Farm

would like to acquire Blackacre and has had the title checked to see who owns Blackacre so it

48

Page 53: FEBRUARY 2016 PENNSYLVANIA BAR … 2016 PENNSYLVANIA BAR EXAMINATION Essay Questions and Examiners’ Analyses and Performance Test Pennsylvania Board of Law Examiners 601 Commonwealth

can approach the owner. The title report indicates that Tony, a lifelong resident of State Y, is the

record owner of Blackacre. Tony died two months ago. His will has been probated in State Y,

and an ancillary estate has been opened in Pennsylvania. Tony’s will specifically devises

Blackacre to Nancy, his niece, a Pennsylvania resident. Tony has one child, Sonny, who is the

residuary beneficiary under Tony’s will.

The title report shows that prior to his death, Tony had sold Blackacre to a third party.

Tony financed the sale to the third party and took back a mortgage to secure the financing.

When the third party defaulted on the loan, Tony foreclosed, and the third party delivered a deed

for Blackacre back to Tony in lieu of foreclosure prior to Tony’s death.

Under State Y law, Tony’s lifetime sale of Blackacre results in an ademption.

Consequently, under State Y law, Blackacre would now be part of Tony’s residuary estate.

Under Pennsylvania law, however, a devisee of specifically devised property has a right to any

of that property owned by the testator at his death even if obtained in lieu of foreclosure on the

security for the specifically devised property. Thus, under Pennsylvania law, there would be no

ademption and Blackacre would pass to Nancy. Nancy and Sonny intend to litigate the right to

ownership of Blackacre in the appropriate Pennsylvania court.

1. (a) If Karen purchases Kitchen and Bob later learns of Karen’s purchase of

Kitchen, under Pennsylvania corporate law, what substantive argument should

Farm assert to challenge Karen’s purchase?

(b) In such a challenge, what remedy(s) might Farm seek?

2. Other than the warranty of good title, what implied warranties are potentially

applicable under the Uniform Commercial Code with respect to the tractor, and

could Farm successfully sue Equipment for breach of these warranties?

3. Assuming the Pennsylvania court where Blackacre is located is the proper forum

to resolve the Blackacre ownership issue, in deciding who should receive

Blackacre, which state’s law should the Pennsylvania court apply, and with what

result?

49

Page 54: FEBRUARY 2016 PENNSYLVANIA BAR … 2016 PENNSYLVANIA BAR EXAMINATION Essay Questions and Examiners’ Analyses and Performance Test Pennsylvania Board of Law Examiners 601 Commonwealth

Question No. 6: Examiner’s Analysis

1. (a) Farm should assert that Karen has breached her duty of loyalty to Farm and

has taken advantage of a corporate opportunity to the detriment of Farm.

The Pennsylvania Business Corporation Law of 1988, as amended, (the “BCL”) states,

“[a] director of a business corporation shall stand in a fiduciary relation to the corporation and

shall perform his duties as a director . . . in good faith, in a manner he reasonably believes to be

in the best interests of the corporation.” 15 Pa. C.S.A. § 1712(a). This is often referred to as the

duty of loyalty. Section 1712(c) of the BCL similarly imposes a duty of loyalty upon corporate

officers. Directors of a corporation have been analogized to trustees of a trust with the

shareholders of the corporation being the beneficiaries of the trust. See, 3 W. EDWARD SELL &

WILLIAM H. CLARK, JR., Pennsylvania Business Corporations, § 1712.3 (Rev. 2d ed. 1997). As

such, “a director acting in good faith will not put herself in a position where her personal

interests conflict with her duty to the corporation.” Id. § 1712.4.

Karen’s acquisition of Kitchen presents a classic “corporate opportunity” situation. “The

law has long recognized the doctrine of corporate opportunity which prohibits one who occupies

a fiduciary relationship to a corporation from acquiring, in opposition to the corporation,

property in which the corporation has an interest or tangible expectancy or which is essential to

its existence.” Fletcher Cyc. Corp., § 861.10 (Perm. ed. 1994). A corporate opportunity is said

to exist “when a proposed activity is reasonably incident to the corporation’s present or

prospective business and is one in which the corporation has the capacity to engage.” Id.

Directors “must devote themselves to the corporate affairs with a view to promote the common

interests and not their own, and they cannot, either directly or indirectly, utilize their position to

obtain any personal profit or advantage other than that enjoyed also by their fellow

shareholders.” Seaboard Industries, Inc. v. Monaco, 442 Pa. 256, 261-62, 276 A.2d 305, 309

(1971) (citations omitted). The Seaboard court further stated:

In short, there is demanded of the officer or director of a corporation that he

furnish to it his undivided loyalty; if there is presented to him a business

opportunity which is within the scope of its own activities and of present or

potential advantage to it, the law will not permit him to seize the opportunity for

himself; if he does so, the corporation may elect to claim all of the benefits of the

transaction.

Id. at 262, 276 A.2d at 309.

Karen had a duty to disclose to Farm (and for all practical matters to Bob) her knowledge

of the fact that Kitchen was for sale. If, after full and fair disclosure of the opportunity to the

corporation, it chose not to avail itself of the opportunity then the director could, assuming no

other conflict exists, pursue the opportunity. SELL & CLARK, supra. § 1712.6. “Thus, the

appropriate method to determine whether or not a corporate opportunity exists is to let the

corporation decide at the time the opportunity is presented.” Fletcher, supra § 861.10. It should

be noted that even if Farm had passed on the opportunity, thereby allowing Karen to purchase

50

Page 55: FEBRUARY 2016 PENNSYLVANIA BAR … 2016 PENNSYLVANIA BAR EXAMINATION Essay Questions and Examiners’ Analyses and Performance Test Pennsylvania Board of Law Examiners 601 Commonwealth

Kitchen, that any contract between Kitchen and Farm would have to pass muster under any

applicable conflict of interest rules due to the fact that Karen would then own a business that is

doing business with another business in which she was a board member. See, SELL & CLARK,

supra § 1712.

The facts indicate that obtaining Kitchen would enhance Farm’s operations and increase

its profits. Farm has the financial capability to purchase Kitchen. Farm has been paying Kitchen

to process its fruit and having its own processing kitchen would likely increase its margins and

profitability. Karen appears to be acting out of spite to Bob and her usurpation of the Kitchen

opportunity flies directly in the face of her duty of loyalty to Farm and its other shareholder,

Bob. Karen should not have taken advantage of the opportunity until she first fully disclosed it

to the board (basically Bob) and if the board had rejected the opportunity, she could then have

proceeded with her purchase of Kitchen. Karen purchased Kitchen without first giving Farm the

opportunity to act, and as a result she breached her duty of loyalty to Farm.

1. (b) Farm most likely will seek the impression of a constructive trust and an

accounting from Karen.

“The corporate opportunity doctrine rests upon the broad foundation of public policy that,

for the purpose of removing all temptation, extinguishes all possibility of profit flowing from a

breach of the confidence imposed by the fiduciary relation.” Fletcher, supra, § 861.50.

Accordingly, a director who breaches her duty of loyalty to a corporation by usurping an

opportunity that properly belongs to the corporation will not be allowed to benefit from her

breach. Id.

The remedies for usurpation of a corporate opportunity will generally lie in equity. See

SELL, supra, § 1712.3. The remedy most often applicable to a misappropriation of corporate

opportunity is the imposition of a constructive trust for the benefit of the corporation upon the

property or opportunity. See, Fletcher, supra, §861.50. A constructive trust is a relationship

with respect to property usually subjecting the title holder to an equitable duty to convey the

property to another who has been wrongfully deprived of the property resulting in unjust

enrichment to the one upon whom the constructive trust is imposed. RESTATEMENT (THIRD) OF

TRUSTS, § 1.e. (AM LAW INST. 2003). “A constructive trust is imposed not necessarily to

effectuate an expressed or implied intention, but [instead seeks] to redress a wrong or to prevent

unjust enrichment.” Id. On occasion, courts have also required the party who has usurped the

corporate opportunity to account for all profits derived because of the usurpation of opportunity

and to pay over those profits for the benefit of the corporation. See Seaboard Industries, Inc. v.

Monaco, 442 Pa. 256, 276 A.2d 305 (1971).

Accordingly, Farm could seek the imposition of a constructive trust upon the Kitchen

opportunity taken by Karen. Farm could also ask the court to compel Karen to account for any

profits derived as a result of her usurpation of the opportunity.

51

Page 56: FEBRUARY 2016 PENNSYLVANIA BAR … 2016 PENNSYLVANIA BAR EXAMINATION Essay Questions and Examiners’ Analyses and Performance Test Pennsylvania Board of Law Examiners 601 Commonwealth

2. The implied warranties of merchantability and fitness for particular purpose could

potentially arise from the sale of the tractor. While only the warranty of

merchantability is breached under the facts, Farm would not be successful in a suit

claiming breach of either warranty because the warranties were disclaimed when

Equipment sold the tractor “as is and with all faults.”

Article II of the Pennsylvania Uniform Commercial Code (the “Code”) generally applies

to transactions involving the sale of goods. 13 Pa. C.S.A. § 2102. Under the Code, “goods”

consist of “all things (including specially manufactured goods) which are movable at the time of

identification to the contract for sale other than money in which the price is to be paid,

investment securities (Division 8) and things in action.” 13 Pa. C.S.A. § 2105(a). Thus, the

tractor sold by Equipment is a “good” under the Code.

Other than the warranty of good title and the warranty against infringement, the Code

generally provides for two implied warranties that might be asserted in connection with the sale

of a good. Section 2314 implies a warranty of merchantability and Section 2315 a warranty of

fitness for particular purpose. The implied warranty of fitness for a particular purpose requires

that “the seller at the time of contracting has reason to know: (1) any particular purpose for

which the goods are required; and (2) that the buyer is relying on the skill or judgment of the

seller to select or furnish suitable goods.” 13 Pa. C.S.A. § 2315. It is unlikely that the implied

warranty of fitness for particular purpose would be implicated under the facts. For this warranty

to apply Equipment would have to have had reason to know the particular purpose for which the

tractor was required by Farm, and that Farm was relying on the skill and judgment of Equipment

to select or furnish a suitable tractor. The facts do not suggest that this was the case and instead

indicate that Equipment did not make any representations regarding the quality or suitability of

the tractor upon which Farm could have relied. Therefore, this warranty would not likely be

applicable.

The implied warranty of merchantability provides in relevant part the following:

(a) Sale by merchant. - Unless excluded or modified (section 2316), a warranty

that the goods shall be merchantable is implied in a contract for their sale if the

seller is a merchant with respect to goods of that kind. . . .

(b) Merchantability standards for goods. - Goods to be merchantable must be at

least such as:

* * *

(3) are fit for the ordinary purposes for which such goods are used.

13 Pa. C.S.A. § 2314.

Equipment clearly is a merchant as the facts state that it sells new and used farm

equipment. A merchant is “[a] person who: (1) deals in goods of the kind; or (2) otherwise by

his occupation holds himself out as having knowledge or skill peculiar to the practices or goods

involved in the transaction or to whom such knowledge or skill may be attributed by his

52

Page 57: FEBRUARY 2016 PENNSYLVANIA BAR … 2016 PENNSYLVANIA BAR EXAMINATION Essay Questions and Examiners’ Analyses and Performance Test Pennsylvania Board of Law Examiners 601 Commonwealth

employment of an agent or broker or other intermediary who by his occupation holds himself out

as having such knowledge or skill.” 13 Pa. C.S.A. § 2104.

The facts state that Farm was only able to use the tractor for two days and the rest of the

three weeks that Farm owned it, the tractor was repeatedly in the shop for various repairs.

Consequently, the tractor when sold does not appear to have been fit for its ordinary purpose,

i.e., use as a tractor. Thus, it appears that this warranty could apply.

Assuming that either of the foregoing warranties applies, the warranty claims should be

barred by the disclaimer that appears in the agreement of sale. The Code addresses the manner

in which a merchant may disclaim implied warranties. See 13 Pa. C.S.A. §2316. Section 2316

provides, inter alia:

(b) Implied warranties of merchantability and fitness.—Subject to subsection (c),

to exclude or modify the implied warranty of merchantability or any part of it the

language must mention merchantability and in case of a writing must be

conspicuous, and to exclude or modify any implied warranty of fitness the

exclusion must be by a writing and conspicuous. Language to exclude all implied

warranties of fitness is sufficient if it states, for example, that “There are no

warranties which extend beyond the description on the face hereof.”

(c) Implied warranties in general. - Notwithstanding subsection (b):

(1) Unless the circumstances indicate otherwise, all implied warranties

are excluded by expressions like “as is,” “with all faults” or other

language which in common understanding calls the attention of the buyer

to the exclusion of warranties and makes plain that there is no implied

warranty.

13 Pa. C.S.A. §2316.

The sales agreement clearly states that the tractor is being sold “as is and with all faults.”

The “as is and with all faults” language is in bold print and appears on the first page of the

agreement. Absent circumstances that would indicate otherwise, none of which appear to be

present, the “as is and with all faults” language should protect the seller from implied warranty

claims. Terms such as those appearing in this contract “are understood to mean that the buyer

takes the entire risk as to the quality of the goods involved.” Thomas M. Quinn, Uniform

Commercial Code Commentary and Law Digest, § 2-316[A][3][c] (1991). Accordingly, any

claims asserted by Farm under these implied warranties are likely to fail because of the

disclaimer language included in the sales agreement.

3. The Pennsylvania court should apply the law of Pennsylvania in determining

whether or not an ademption has occurred, which will result in a conclusion that no

ademption has occurred, and title to Blackacre will pass to Nancy under the will.

53

Page 58: FEBRUARY 2016 PENNSYLVANIA BAR … 2016 PENNSYLVANIA BAR EXAMINATION Essay Questions and Examiners’ Analyses and Performance Test Pennsylvania Board of Law Examiners 601 Commonwealth

The facts present a conflict of laws issue regarding the disposition of real property

located in Pennsylvania under a will of a State Y testator. The will was probated in State Y.

Under State Y law, Tony’s conveyance of Blackacre during his lifetime would affect an

ademption thus depriving Nancy, the specific devisee of Blackacre, of her devise upon Tony’s

death. Under Pennsylvania law, a specific devisee can essentially follow the devised property

conveyed during the lifetime of the testator or can follow a mortgage or re-conveyance by deed

in lieu of foreclosure. See, 20 Pa. C.S.A. § 2514 (18). Under Pennsylvania law Nancy would be

entitled to receive Blackacre.

Generally, questions regarding the disposition of land, as affected by interpretation of

wills, are governed by the law of the state where the land is located. The Restatement of Conflict

of Laws provides with respect to construction of a will devising land, “In the absence of [a

designation to the contrary in the will], the will is construed in accordance with the rules of

construction that would be applied by the courts of the situs.” RESTATEMENT (SECOND) OF

CONFLICT OF LAWS § 240(2) (1971). The Pennsylvania Supreme Court has stated:

[I]t is a principle of private, international law, fortified by a great mass of

authority, that all questions relating to the transfer of title to land wherever arising

will be governed by the laws of the place where the land is situated. . . . This

principle is applicable to questions relating to the effect of language in wills of

testators not domiciled in the dominant situs. . . . Specifically, whether the sale of

devised land operates as an ademption is governed by the law of the situs.

In re Dublin’s Estate, 375 Pa. 599, 603-04, 101 A.2d 731, 733 (1954) (citations omitted). “The

situs state of realty is generally entitled to the severest deference. . . . That the laws of the situs

state should govern the devise of real property is a sound principle, articulated in both

Restatements of Conflict of Laws, and in the consistent statements of this Court.” In re Estate of

Janney, 498 Pa. 398, 401-02, 446 A.2d 1265, 1266 (1982). The situs of the land rule is widely

recognized when title to land is involved. Id.

The court should apply the situs rule set forth above. Pennsylvania has the greatest

interest in regulating the transfer of title to land within its boundaries. Predictability and

certainty in matters of title transference are paramount in all states. Id.

Given the situs rule, the Pennsylvania court should apply Pennsylvania law, which will

have the effect of finding that an ademption has not occurred. Accordingly, Nancy should

receive Blackacre.1

1 If one were to be unaware of the “situs” rule and would analyze this issue using the general Restatement principal

that a state should apply the local law of the state which has the most significant relationship to the thing or the

parties the result should be the same. RESTATEMENT (SECOND) OF CONFLICT OF LAWS § 222 (1971). The land in

question is located in Pennsylvania and the courts of that state clearly have an interest in predictability relative to

transfers of property within the state. The Restatement of Conflict of Laws observes “both the convenience of title

searchers and the security of titles would be served if the rules of construction of the situs were always applicable.

Otherwise, there would be situations, perhaps uncertain both in their nature and number, where the rules of

construction of one or more other states would have to be consulted.” RESTATEMENT (SECOND) OF CONFLICTS OF

LAWS, 240(2), cmt. f.

54

Page 59: FEBRUARY 2016 PENNSYLVANIA BAR … 2016 PENNSYLVANIA BAR EXAMINATION Essay Questions and Examiners’ Analyses and Performance Test Pennsylvania Board of Law Examiners 601 Commonwealth

Question No. 6: Grading Guidelines

1. (a) Corporate opportunity doctrine

Comments: Candidates should recognize that usurpation of a corporate opportunity violates a

director’s duty of loyalty to the corporation and that to comply with the duty the director should

have recognized that the opportunity would benefit the corporation and should have disclosed it

to the corporation.

5 points

1. (b) Remedies for usurpation of corporate opportunity

Comments: Candidates should discuss the equitable remedies of impression of a constructive

trust and an accounting to allow the corporation to receive the benefit of the opportunity and any

profits.

4 points

2. Implied warranties under the Uniform Commercial Code—Sales and disclaimer

Comments: Candidates should identify the two implied warranties that might arise under the

Uniform Commercial Code and briefly explain each. The candidates should then discuss the

existence of the conspicuous disclaimer and the effect of the disclaimer upon any claims for

breach of implied warranty.

7 points

3. Choice of law

Comments: The candidates should recognize that this is a conflict of law issue. They should

recognize that it involves a devise of real property and discuss how the conflict rules might be

applied.

4 points

55

Page 60: FEBRUARY 2016 PENNSYLVANIA BAR … 2016 PENNSYLVANIA BAR EXAMINATION Essay Questions and Examiners’ Analyses and Performance Test Pennsylvania Board of Law Examiners 601 Commonwealth

PT

Supreme Court of Pennsylvania

Pennsylvania Board of Law Examiners

Pennsylvania Bar Examination

February 24 and 25, 2015

PERFORMANCE TEST

February 24, 2015

Use GRAY covered book for your answer to the Performance Test.

© 2015 Pennsylvania Board of Law Examiners

PLACE BAR CODED APPLICANT LABEL HERE

Question Number 3

on CBT Software

56

Page 61: FEBRUARY 2016 PENNSYLVANIA BAR … 2016 PENNSYLVANIA BAR EXAMINATION Essay Questions and Examiners’ Analyses and Performance Test Pennsylvania Board of Law Examiners 601 Commonwealth

TABLE OF CONTENTS

FILE

Memorandum to applicant outlining task ........................................................................................1

Complaint .........................................................................................................................................2

Affidavit of Emily Jaeger .................................................................................................................3

LIBRARY

Statute

42 Pa. C.S. §5322 .............................................................................................................................4

Case law

Mendel v. Williams...........................................................................................................................5

57

Page 62: FEBRUARY 2016 PENNSYLVANIA BAR … 2016 PENNSYLVANIA BAR EXAMINATION Essay Questions and Examiners’ Analyses and Performance Test Pennsylvania Board of Law Examiners 601 Commonwealth

FILE

58

Page 63: FEBRUARY 2016 PENNSYLVANIA BAR … 2016 PENNSYLVANIA BAR EXAMINATION Essay Questions and Examiners’ Analyses and Performance Test Pennsylvania Board of Law Examiners 601 Commonwealth

MEMORANDUM

TO: Applicant

FROM: Jean Kerwick, Managing Partner

CLIENT: Esther Energy Industries, Client # 062470

DATE: February 23, 2016

SUBJECT: Assignment to Draft Legal Memorandum

We represent Esther Energy Industries (EEI), which has just been sued in Pennsylvania court by a

disgruntled, recently terminated manager, Margaret Anderson. In her complaint, Ms. Anderson is

seeking damages from EEI for wrongful discharge. See Complaint. I intend to file preliminary

objections to have the Complaint dismissed based on a lack of personal jurisdiction, among other

grounds.

I have already drafted an argument concluding that there is no basis for finding that general

jurisdiction exists against EEI and have identified the relevant research to support an argument that

specific jurisdiction against EEI is also not available. Your assignment is to draft a persuasive

memorandum of law setting forth all arguments that can be advanced to support the conclusion that

the court lacks specific personal jurisdiction over EEI. You should raise arguments that can be

advanced to support our position in the alternative even if inconsistent with a prior argument. Your

argument should reference the applicable standards for ruling on preliminary objections based on a

lack of personal jurisdiction and apply the relevant legal principles to the facts to demonstrate the

reasoning that supports your conclusion.

Your legal memorandum will assist me in drafting a brief in support of preliminary objections to

Anderson’s Complaint. While other issues exist that could serve as a basis for preliminary

objections, you are to limit your memorandum to the issues identified above.

Attached is an affidavit from the Corporate Secretary of EEI. Our client will execute this affidavit

and we will attach it to our preliminary objections. The facts set forth in the affidavit are

uncontroverted and may be used in drafting your memorandum.

Authority

My research includes Pennsylvania’s Long Arm Statute and the attached relevant excerpts of an

applicable case, which you should rely upon when drafting your memorandum. Do not rely upon

your personal knowledge of these issues, or on cases and statutes that are not included in the

attached File and Library. Instead, you should base your analysis and conclusions only upon the

documents provided in the File and the Library.

Format

Please make sure your document is in the form of a legal memorandum, with a header similar to the

header on this page. Insert in the subject line “Legal Memorandum,” address your memorandum to

me, and list yourself as “Applicant” in the “From” line. Formal Bluebook citations are not required;

however, you must sufficiently identify each document, whether it be the case, statute, complaint, or

affidavit such that I will know to what document you are referring in your memorandum.

59

Page 64: FEBRUARY 2016 PENNSYLVANIA BAR … 2016 PENNSYLVANIA BAR EXAMINATION Essay Questions and Examiners’ Analyses and Performance Test Pennsylvania Board of Law Examiners 601 Commonwealth

Margaret Anderson, Plaintiff : IN THE COURT OF COMMON PLEAS

:

v. : OF FENDICK COUNTY

:

Esther Energy Industries, Defendant : CIVIL ACTION – LAW

: NO. 631995 of 2016

COMPLAINT

AND NOW, comes Margaret Anderson, by and through her attorney, and respectfully

states to this Honorable Court:

1. The Plaintiff is Margaret Anderson, who resides at 1 Naysayer Street, Smith City,

Fendick County, Pennsylvania (Ms. Anderson is hereinafter referred to as “Plaintiff”).

2. The Defendant is Esther Energy Industries, a Delaware corporation that owns

property in the Commonwealth of Pennsylvania (hereinafter referred to as “Defendant”).

3. Defendant’s property interests in Pennsylvania constitute continuous and

substantial activities that have been and are carried out in Pennsylvania.

4. At all times material hereto, the Plaintiff was an employee of Defendant.

5. Plaintiff’s primary duties included overseeing the rental operations of Defendant’s

building in Pennsylvania, including reviewing leases, billing tenants, and depositing rent checks

received from tenants of the building in Pennsylvania. Plaintiff also visited the facility in

Pennsylvania on two occasions: once to sign a lease, and on another occasion to inspect storm

damage to the building.

6. Plaintiff was called to serve on jury duty in County Y in Oklahoma beginning on

Monday, December 28, 2015, and was forced to miss the entire week of work to perform her

service as a juror.

7. On Monday, January 4, 2016, Plaintiff drove to her office in Oklahoma, at which

time she was informed by her manager that she had been fired.

8. Plaintiff was terminated without legitimate cause; and was fired for serving on a

jury, which was required under Oklahoma law.

9. Due to her wrongful termination, Plaintiff is entitled to be compensated.

WHEREFORE, the Plaintiff prays this Honorable Court enter judgment in Plaintiff’s

favor in an amount exceeding the arbitration limits applicable to her claim.

Respectfully Submitted:

By: Maria Myers Maria Myers, Counsel to Plaintiff

60

Page 65: FEBRUARY 2016 PENNSYLVANIA BAR … 2016 PENNSYLVANIA BAR EXAMINATION Essay Questions and Examiners’ Analyses and Performance Test Pennsylvania Board of Law Examiners 601 Commonwealth

Margaret Anderson, Plaintiff : IN THE COURT OF COMMON PLEAS

v. : OF FENDICK COUNTY

:

Esther Energy Industries, Defendant : CIVIL ACTION – LAW

: NO. 631995 of 2016

AFFIDAVIT OF EMILY JAEGER

I, Emily Jaeger, as Corporate Secretary for Defendant, Esther Energy Industries (EEI), do

hereby swear and affirm, subject to the penalties of 18 Pa.C.S. § 4904, relating to unsworn

falsification to authorities, that:

1. EEI is a company incorporated in Delaware, and which is engaged primarily in the storage

and distribution of natural gas.

2. EEI makes in excess of $1 billion per year from its gas operations.

3. In 2012, EEI purchased a building in Fendick County, Pennsylvania, with the intent to use it

as its corporate headquarters and principal place of business. Before it could ever occupy the

building, or move anything or anyone into the building, EEI was purchased by another

company in Oklahoma, and moved its headquarters there.

4. Since mid-2012, EEI has and continues to advertise on its website for sale the building in

which its headquarters was to be located in Pennsylvania, but no buyers have emerged.

5. Although the property remains for sale, in January 2015, EEI rented the entire building to a

frozen yogurt manufacturer for $14,000 per year to offset some of its costs.

6. Ownership of the building is the only connection EEI has with Pennsylvania.

7. EEI does not regularly conduct business in Pennsylvania.

8. EEI was never registered to do business in Pennsylvania; and EEI was never qualified as a

foreign corporation under the laws of the Commonwealth of Pennsylvania.

9. EEI does not, and has never, stored documents in Pennsylvania.

10. EEI does not, and has never, employed anyone in Pennsylvania.

11. EEI does not, and has never, purchased products in or from Pennsylvania.

12. EEI does not utilize the services of a financial institution in Pennsylvania; instead, all rent

checks derived from its owned property are mailed to Oklahoma for deposit.

13. After she was terminated, Margaret Anderson moved to Pennsylvania.

Emily Jaeger Emily Jaeger

Corporate Secretary

Esther Energy Industries

61

Page 66: FEBRUARY 2016 PENNSYLVANIA BAR … 2016 PENNSYLVANIA BAR EXAMINATION Essay Questions and Examiners’ Analyses and Performance Test Pennsylvania Board of Law Examiners 601 Commonwealth

LIBRARY

62

Page 67: FEBRUARY 2016 PENNSYLVANIA BAR … 2016 PENNSYLVANIA BAR EXAMINATION Essay Questions and Examiners’ Analyses and Performance Test Pennsylvania Board of Law Examiners 601 Commonwealth

42 Pa.C.S. § 5322 Bases of personal jurisdiction over persons outside this Commonwealth

(a) General rule.--A tribunal of this Commonwealth may exercise personal jurisdiction over a

person . . . who acts directly or by an agent, as to a cause of action or other matter arising from

such person:

(1) Transacting any business in this Commonwealth. Without excluding other acts

which may constitute transacting business in this Commonwealth, any of the following shall

constitute transacting business for the purpose of this paragraph:

(i) The doing by any person in this Commonwealth of a series of similar acts for the

purpose of thereby realizing pecuniary benefit or otherwise accomplishing an object.

(ii) The doing of a single act in this Commonwealth for the purpose of thereby

realizing pecuniary benefit or otherwise accomplishing an object with the intention of

initiating a series of such acts.

(iii) The shipping of merchandise directly or indirectly into or through this

Commonwealth.

(iv) The engaging in any business or profession within this Commonwealth, whether

or not such business requires license or approval by any government unit of this

Commonwealth.

(v) The ownership, use or possession of any real property situate within this

Commonwealth.

(2) Contracting to supply services or things in this Commonwealth.

(3) Causing harm or tortious injury by an act or omission in this Commonwealth.

(4) Causing harm or tortious injury in this Commonwealth by an act or omission outside

this Commonwealth.

(5) Having an interest in, using, or possessing real property in this Commonwealth.

* * *

(b) Exercise of full constitutional power over nonresidents. In addition to the provisions of

subsection (a) the jurisdiction of the tribunals of this Commonwealth shall extend to all persons

. . . to the fullest extent allowed under the Constitution of the United States and may be based on

the most minimum contact with this Commonwealth allowed under the Constitution of the

United States.

(c) Scope of jurisdiction.--When jurisdiction over a person is based solely upon this section,

only a cause of action or other matter arising from acts enumerated in subsection (a) or from acts

forming the basis of jurisdiction under subsection (b), may be asserted against him.

* * *

63

Page 68: FEBRUARY 2016 PENNSYLVANIA BAR … 2016 PENNSYLVANIA BAR EXAMINATION Essay Questions and Examiners’ Analyses and Performance Test Pennsylvania Board of Law Examiners 601 Commonwealth

April Mendel, Appellant

v.

Eric Williams, M.D., et al.

* * *

I. Factual History

April Mendel appeals from the March

17, 2011 and March 21, 2011 orders of the

Court of Common Pleas of Philadelphia County,

sustaining the preliminary objections filed by

Underwood Memorial Hospital (“Underwood”)

and Doctor Robert Ocasio, M.D. asserting lack

of personal jurisdiction in Pennsylvania. . . . On

July 31, 2008, Eric Williams, M.D. and Andrew

Beaver, M.D. performed L3-S1 laminectomy

surgery on Mendel's spine at the Albert Einstein

Medical Center ("Einstein") in Philadelphia,

Pennsylvania. Doctor Williams and Doctor

Beaver are licensed to practice medicine in

Pennsylvania; both maintain a medical office

and regular place of business at Einstein.

Mendel left Einstein on August 4, 2008,

and returned to her home in Turnersville, New

Jersey. The following week, on August 11,

2008, Mendel experienced drainage from her

surgical wound and contracted a fever. Mendel

contacted a member of Doctor William's staff

who suggested that she go to an emergency

room. Mendel went to Underwood in

Woodbury, New Jersey later that day.

Underwood emergency room physicians

confirmed that Mendel had a wound infection in

the laminectomy incision with purulent

drainage. The emergency room physicians

contacted Doctor Williams, who agreed to

accept Mendel at Einstein, but stated that there

would not be an available bed until the

following day. In the interim, Mendel was

admitted to the internal medicine service of

Doctor Ocasio at Underwood.

During the course of the night, Mendel

complained of worsening pain in her legs.

Doctor Ocasio and other Underwood physicians

approved increasingly strong pain medication,

but failed to diagnose and treat the epidural

abscess that was compressing Mendel's spine.

Mendel further alleges that Doctor Ocasio did

not make any mention of Mendel's pain in her

discharge summary, describing her condition as

"stable," and that he certified her transfer to

Einstein without warning Doctor Williams of

her worsening condition.

Mendel was transported by ambulance to

Einstein the following morning, on August 12,

2008. Doctor Williams performed additional

surgery later that day at Einstein to correct the

infected wound. The surgery revealed that the

abscess extended to the spinal cord and that

Mendel suffered paralysis below the waist.

Doctors subsequently discharged Mendel to a

rehabilitation center on August 29, 2008.

Despite rehabilitation efforts, Mendel has not

regained movement or feeling below the waist.

64

Page 69: FEBRUARY 2016 PENNSYLVANIA BAR … 2016 PENNSYLVANIA BAR EXAMINATION Essay Questions and Examiners’ Analyses and Performance Test Pennsylvania Board of Law Examiners 601 Commonwealth

II. Procedural History

Mendel initiated the instant action by

writ of summons in the Court of Common Pleas

of Philadelphia County . . . against Underwood,

Doctor Ocasio, Einstein and the treating

physicians at Einstein.

* * *

Mendel filed a complaint on August 6,

2010. Doctor Ocasio filed preliminary

objections to the complaint on August 13, 2010,

alleging lack of personal jurisdiction.

. . . On March 17, 2011, the trial court entered

an order dismissing Mendel's action against

Doctor Ocasio for lack of personal jurisdiction

in Pennsylvania.

Underwood filed preliminary objections

to Mendel's complaint on September 14, 2011,

also alleging Pennsylvania lacked personal

jurisdiction.

. . . On March 21, 2011, the trial court entered

its order dismissing the action against

Underwood for lack of personal jurisdiction.

Mendel filed a timely appeal.

* * *

III. Analysis

A. Personal Jurisdiction

Mendel avers that the trial court erred in

granting Doctor Ocasio and Underwood's

preliminary objections as to personal

jurisdiction in Pennsylvania.

[The] standard of review [for]

granting preliminary objections

challenging personal jurisdiction

is as follows: [W]hen deciding a

motion to dismiss for lack of

personal jurisdiction the court

must consider the evidence in the

light most favorable to the non-

moving party. . . . Once the

moving party supports its

objections to personal

jurisdiction, the burden of

proving personal jurisdiction is

upon the party asserting it.

Courts must resolve the question

of personal jurisdiction based on

the circumstances of each

particular case.

Schiavone v. Aveta, 41 A.3d 861, 865 (Pa.Super.

2012) (citation omitted).

* * *

The Due Process Clause of the

Fourteenth Amendment to the United States

Constitution limits the authority of a state to

exercise in personam jurisdiction over non-

resident defendants. Burger King Corp. v.

Rudzewicz, 471 U.S. 462, 471-72, 105 S.Ct.

2174, 85 L.Ed.2d 528 (1985). The extent to

which jurisdiction is proscribed by the Due

Process Clause is dependent upon the nature and

quality of the defendant's contacts with the

forum state. See id. at 474-76 . . . ; Kubik v.

Letteri, 532 Pa. 10 . . . (1992). Where a

defendant "has established no meaningful

contacts, ties or relations" with the forum, the

Due Process Clause prohibits the exercise of

personal jurisdiction. Burger King, supra at

472. However, where a defendant has

65

Page 70: FEBRUARY 2016 PENNSYLVANIA BAR … 2016 PENNSYLVANIA BAR EXAMINATION Essay Questions and Examiners’ Analyses and Performance Test Pennsylvania Board of Law Examiners 601 Commonwealth

"purposefully directed" his activities at the

residents of the forum, he is presumed to have

"fair warning" that he may be called to suit

there. Id.

A defendant's activities in the forum

State may give rise to either specific jurisdiction

or general jurisdiction. See Kubik, supra at

1113. "Specific jurisdiction

. . . depends on an 'affiliatio[n] between the

forum and the underlying controversy,'

principally, activity or an occurrence that takes

place in the forum State and is therefore subject

to the State's regulation." Goodyear Dunlop

Tires Operations, S.A. v. Brown, ___U.S. ___,

131 S.Ct. 2846 . . . (2011). Because due

process may permit specific jurisdiction based

solely on "single or occasional" acts

purposefully directed at the forum, it is narrow

in scope, limiting a cause of action to the extent

that it "arises out of or relates to" the very

activity that establishes jurisdiction. See id. at

2851.

* * *

2. Specific Jurisdiction over Underwood and

Doctor Ocasio

A foreign defendant who does not have

sufficient contacts with Pennsylvania to

establish general jurisdiction may nevertheless

be subject to specific jurisdiction in

Pennsylvania pursuant to the Pennsylvania

Long-Arm Statute, 42 Pa.C.S.A. § 5322 (Bases

of personal jurisdiction over persons outside this

Commonwealth). Section 5322(a) contains ten

paragraphs that specify particular types of

contact with Pennsylvania deemed sufficient to

warrant the exercise of specific jurisdiction.

Scoggins v. Scoggins, 382 Pa.Super. 507 . . .

(1989); 42 Pa.C.S.A. § 5322(a). . . . [I]f a

defendant's activities in Pennsylvania only give

rise to jurisdiction under section 5322(a) or (b),

the plaintiff's cause of action is limited to those

activities which formed the basis of jurisdiction.

See 42 Pa.C.S.A. § 5322(c).

Once it is determined that jurisdiction is

authorized by the Long-Arm Statute, the party

seeking relief must demonstrate that the exercise

of jurisdiction conforms with the Due Process

Clause. . . . (two requirements necessary for

specific jurisdiction in Pennsylvania: (1)

jurisdiction must be authorized by state Long-

Arm Statute; and (2) jurisdiction must comport

with constitutional principles of due process).

Whether specific jurisdiction is proper under the

Due Process Clause requires a two-part analysis:

first, the plaintiff must demonstrate that the

defendant purposefully established minimum

contacts with the forum state; and second, the

maintenance of the suit must not offend

"traditional notions of fair play and substantial

justice." See Schiavone, supra at 869. . . .

A defendant purposefully establishes

minimum contacts with the forum state when its

66

Page 71: FEBRUARY 2016 PENNSYLVANIA BAR … 2016 PENNSYLVANIA BAR EXAMINATION Essay Questions and Examiners’ Analyses and Performance Test Pennsylvania Board of Law Examiners 601 Commonwealth

contacts are: such that the defendant could

reasonably anticipate being called to defend

itself in the forum. . . .Random, fortuitous, and

attenuated contacts cannot reasonably notify a

party that it may be called to defend itself in a

foreign forum and, thus, cannot support the

exercise of personal jurisdiction. That is, the

defendant must have purposefully directed its

activities to the forum and conducted itself in a

manner indicating that it has availed itself of the

forum's privileges and benefits such that it

should be subjected to the forum state's laws

and regulations. Aventis Pasteur, Inc. v. Alden

Surgical Co., Inc., 848 A.2d 996, 1000 (Pa.

Super. 2004).

If the defendant has purposefully

established minimum contacts in the forum

State, these contacts must be considered on a

case-by-case basis to determine whether they

"are such as to make it reasonable and fair to

require him to conduct his defense in the state."

Kubik, supra at 1114 . . . .

Factors to be considered include

(1) the burden on the defendant,

(2) the forum state's interest in

adjudicating the dispute, (3) the

plaintiff's interest in obtaining

convenient and effective relief,

(4) the interstate judicial system's

interest in obtaining the most

efficient resolution of

controversies and (5) the shared

interest of the several states in

furthering fundamental

substantive social policies.

Kubik, supra at 1114.

a. Pennsylvania Long-Arm Statute

Mendel asserts that specific jurisdiction

over Underwood and Doctor Ocasio is proper in

Pennsylvania under section 5322(a)(4) of the

Long-Arm Statute. That section provides:

(a) General rule. A tribunal of

this Commonwealth may

exercise personal jurisdiction

over a person . . . who acts

directly or by an agent, as to a

cause of action or other matter

arising from such person:

* * *

(4) Causing harm or tortious

injury in this Commonwealth by

an act or omission outside this

Commonwealth.

42 Pa.C.S.A. § 5322(a)(4).

Mendel claims that Underwood and

Doctor Ocasio meet the statutory requirements

of section 5322(a)(4) because their failure to

diagnose and treat her spinal cord injury in New

Jersey, or to send Einstein a report of her

medical condition after she had been transferred

to Pennsylvania caused her paralysis in

Pennsylvania. Mendel's attempt to frame her

paralysis as a harm or injury caused in

Pennsylvania is facially appealing, but

ultimately unpersuasive.

In . . . Kurtz v. Draur [434 F. Supp. 958

(E.D. Pa. 1977)], the Courts for Pennsylvania

did consider a similar question in applying the

predecessor to the current Long-Arm Statute[,

67

Page 72: FEBRUARY 2016 PENNSYLVANIA BAR … 2016 PENNSYLVANIA BAR EXAMINATION Essay Questions and Examiners’ Analyses and Performance Test Pennsylvania Board of Law Examiners 601 Commonwealth

42 Pa.C.S. § 8305]. Thus, we will look to [this

case] for guidance.

* * *

In Kurtz, a patient sought treatment at a

Nebraska hospital for a heart condition. The

complaint alleged that the treating physician

failed to properly diagnose the condition or to

provide adequate follow-up care. The patient

later moved to Pennsylvania where he suffered

"residual harm" from the alleged negligent

treatment. [T]he Eastern District Court

reasoned that section 8305 "was not designed to

take cognizance of the residual effects of out-of-

state injuries" resulting from medical treatment,

and that "[t]he ‘harm' which [the treating

physician was] alleged to have caused, occurred

in Nebraska." Id. at 961-62.

We agree with the reasoning of the

Court[] in . . . Kurtz, and conclude that in the

instant action, the alleged negligence of

Underwood and Doctor Ocasio did not "[cause]

harm or tortious injury in this Commonwealth"

as contemplated by section 5322(a)(4). See . . .

Kurtz, supra. The mere fact that Mendel's

paralysis was discovered in Pennsylvania, or

that it manifested in Pennsylvania, does not

necessarily mean that it was caused in

Pennsylvania.

Accepting Mendel's factual averments as

true, Underwood and Doctor Ocasio contributed

to Mendel's paralysis by delaying treatment of

her spinal abscess - either through their failure

to treat her injury themselves, or by failing to

notify doctors at Einstein of her status. This

delay began when Mendel was a patient at

Underwood's New Jersey facility. Thus, it

follows that any harm resulting from the delay

was also caused in New Jersey. That the harm

may have continued in Pennsylvania and was

ultimately discovered in Pennsylvania does not

alter the fact that it originated in New Jersey.

* * *

Therefore, we conclude that section

5322(a)(4) of the Long-Arm Statute does not

provide a basis for specific personal jurisdiction

over Underwood or Doctor Ocasio.

b. Due Process

Even if section 5322(a) did encompass

the activities of Underwood and Doctor Ocasio,

specific jurisdiction in Pennsylvania would

nevertheless be prohibited because Underwood

and Doctor Ocasio do not have sufficient

minimum contacts with Pennsylvania to satisfy

the requirements of Due Process. See Derman

v. Wilair Servs., Inc., [404 Pa. Super 136

(1991)] . . . .

Courts have generally been reluctant to

extend specific personal jurisdiction to out-of-

state medical providers for causing injury to

Pennsylvania patients, even though the effects

of the doctors' negligence may be felt in

68

Page 73: FEBRUARY 2016 PENNSYLVANIA BAR … 2016 PENNSYLVANIA BAR EXAMINATION Essay Questions and Examiners’ Analyses and Performance Test Pennsylvania Board of Law Examiners 601 Commonwealth

Pennsylvania. See, e.g., Lebkuecher v.

Loquasto, 255 Pa. Super. 608 . . . (1978).

* * *

In Prince [v. Urban, 49 Cal. App. 4th

1056, 47 Cal. Rptr. 2d 181 (1996)], the

California Fourth District Court of Appeal

considered the limits of personal jurisdiction

over an out-of-state doctor who causes injury to

an in-state patient. There, a California patient

was referred to an Illinois headache specialist

who treated the patient in Illinois for migraines

and prescribed her various medications. After

returning to California, the patient maintained

telephone contact with the specialist and the

specialist continued to prescribe and mail

additional medication to the patient. The patient

subsequently had an adverse reaction to the

medications in California and filed a

malpractice suit against the specialist there.

The Court of Appeal acknowledged that

"the case [was] a close one" for personal

jurisdiction, but concluded that "the balance

[was] tipped in the direction of no jurisdiction"

because of the personal nature of the services

rendered. Id. at 1059. . . . The court held,

"where . . . the out-of-state doctor's contact with

the forum state consists of nothing more than

telephonic follow-up [care] on services rendered

in the doctor's own state, it is unreasonable for

the patient's home state to exercise personal

jurisdiction over the physician." Id. at 1059. . . .

In support, the court noted three factors

in particular which tended to negate jurisdiction.

First, the specialist did not engage in any

"systematic or continuing effort" to provide

services in California. Although the specialist

did offer "follow-up care" to the patient in

California, and the patient suffered injury in

California, the court concluded that the injury

could not be said to have been caused in

California as the treatment was merely a

continuation of one set of services which

originated in Illinois. See id. at 1066. . . .

Second, "the services rendered by [the

specialist] were not 'grounded' in any

relationship with California" because the patient

sought the specialist on her own through a

referral. Id. at 1064 . . . . Finally, recognizing

the policy considerations the court concluded

that limiting jurisdiction would further the

interests of California by encouraging out-of-

state doctors to provide appropriate medical

treatment to California residents, especially

"where specialized medical treatment is needed

and California citizens must travel out of state to

find it." See id. at 1064.

* * *

The facts of the instant case do not

present any of the special characteristics of a

doctor-patient relationship that would justify

extending jurisdiction in Pennsylvania to

Underwood or Doctor Ocasio. . . . Underwood

69

Page 74: FEBRUARY 2016 PENNSYLVANIA BAR … 2016 PENNSYLVANIA BAR EXAMINATION Essay Questions and Examiners’ Analyses and Performance Test Pennsylvania Board of Law Examiners 601 Commonwealth

and Doctor Ocasio did not promote their

practice through national marketing, did not

solicit Mendel as a patient and did not

purposefully treat patients in Pennsylvania. . . .

Additionally, . . . Underwood and Doctor

Ocasio did not maintain any contact with

Mendel after she had returned to Pennsylvania

such that they could be said to have committed a

negligent act here or purposefully directed their

activities here. To the contrary, Mendel's

complaint centers on Underwood and Doctor

Ocasio's failure to act after she returned to

Pennsylvania.

* * *

In fact, it is hard to identify any

purposeful contact that Underwood or Doctor

Ocasio had with Pennsylvania regarding the

treatment of patients. While Mendel lists

numerous activities by Underwood and Doctor

Ocasio in Pennsylvania, including affiliations

with medical organizations in Pennsylvania,

associations with Jefferson and personal travel

to Philadelphia, none of these activities relate to

the injury that Mendel suffered. Accordingly,

they cannot be used as a basis for specific

jurisdiction in the instant action. See 42

Pa.C.S.A. § 5322(c).

* * *

Because Underwood and Doctor Ocasio

could not have reasonably anticipated being

called to court in Pennsylvania based on their

treatment of Mendel in New Jersey, they cannot

be said to have purposefully established

minimum contacts with Pennsylvania. See

Burger King, supra, Aventis Pasteur, supra.

Accordingly, the trial court did not err in

determining that it lacked specific jurisdiction

over Underwood and Doctor Ocasio.

* * *

Mendel has failed to establish that the

exercise of personal jurisdiction over the out-of-

state medical providers in the instant action

would comport with constitutional principles of

due process. . . . [N]either Underwood nor

Doctor Ocasio has purposefully established

minimum contacts in this Commonwealth such

that Pennsylvania may exercise specific

jurisdiction regarding their alleged negligent

treatment of Mendel. See Aventis Pasteur,

supra. Accordingly, the trial court did not err in

sustaining Underwood and Doctor Ocasio's

preliminary objections.

* * *

70

Page 75: FEBRUARY 2016 PENNSYLVANIA BAR … 2016 PENNSYLVANIA BAR EXAMINATION Essay Questions and Examiners’ Analyses and Performance Test Pennsylvania Board of Law Examiners 601 Commonwealth

Instructions

The performance test is designed to test an applicant’s ability to perform the

legal task that has been assigned using the factual information contained in the

File and legal principles that are provided in the Library.

The File contains the only factual information that you should consider in

performing the assigned task. The task to be completed is set forth in the first

document in the File in the form of a memorandum to the applicant. The

Library contains the only legal principles that you should consider to complete

the assigned task. Although your general knowledge of the law may provide

some background for analyzing the problem, the factual information contained

in the File and the legal principles contained in the Library are the only

materials that you should use in formulating your answer to the assigned task.

Your response should be written in the gray answer book or typed in answer

screen number 3 of SofTest. Be sure to allow sufficient time for reading the

materials, organizing your answer and completing the task assigned. Your

answer should demonstrate an understanding of the relevant facts, recognition

of the issues and the applicable principles of law and the reasoning that

supports your answer. Your grade will be based on the content of your

response and your ability to follow instructions in performing the assigned

task.

The events depicted and the persons portrayed by the information in the File

are fictitious and such information does not depict, nor is it intended to depict,

or portray any actual person, company, or occurrence. Any similarity to any

person, living or dead, or any occurrence is purely coincidental.

71

Page 76: FEBRUARY 2016 PENNSYLVANIA BAR … 2016 PENNSYLVANIA BAR EXAMINATION Essay Questions and Examiners’ Analyses and Performance Test Pennsylvania Board of Law Examiners 601 Commonwealth

Question No. PT: Examiner’s Analysis and Grading Guidelines

The applicant is assigned to draft a persuasive legal memorandum to the managing partner to be

used as a basis for a brief in support of preliminary objection due to lack of specific personal

jurisdiction.

The applicant must identify the relevant facts from the Complaint and an Affidavit from the

client’s corporate secretary. The applicant must apply a statute and excerpts from one case to the facts

in order to properly analyze whether specific jurisdiction exists, including any due process arguments.

Format 1 point

Following directions, including formatting directions, is an important skill of every lawyer. The

applicant is expected to follow the directions provided concerning the format of the memorandum (i.e. –

craft their document in the form of a memorandum directed to the managing partner).

The applicant is also expected to provide some form of citation to the complaint, affidavit,

statute, and/or case to sufficiently identify the authority upon which the applicant is relying for each

legal and factual point. Formal Bluebook citations are not required.

The cause of action did not arise from EEI transacting business in Pennsylvania. 8 points

“When deciding a motion to dismiss for lack of personal jurisdiction the court must consider the

evidence in the light most favorable to the non-moving party.” Mendel v. Williams

“Once the moving party supports its objections to personal jurisdiction, the burden of proving

personal jurisdiction is upon the party asserting it.” Mendel v. Williams

A defendant’s activities in the forum state may give rise to either specific jurisdiction or general

jurisdiction. Mendel v. Williams

“A foreign defendant … may … be subject to specific jurisdiction in Pennsylvania pursuant to

the Pennsylvania Long-Arm Statute, 42 Pa.C.S. §5322.” Mendel v. Williams

Specific Jurisdiction depends on an affiliation between the forum and the underlying

controversy, principally, activity or an occurrence that takes place in the forum state and is therefore

subject to the state’s regulation. Mendel v. Williams.

The Pennsylvania Long Arm Statute sets forth ten particular types of contact with Pennsylvania

that are deemed sufficient to warrant the exercise of specific jurisdiction. The only such conduct

applicable to EEI is the ownership, use or possession of any real property situate within this

Commonwealth (42 Pa.C.S. §5322(a)(1)(v)), or if the foreign defendant has an interest in, uses, or

possesses real property in this Commonwealth (42 Pa.C.S. §5322(a)(5)). None of the other transactions

apply.

Pennsylvania’s Long-Arm Statute deems personal jurisdiction to exist if, inter alia, a foreign

defendant has transacted business within the Commonwealth, including the ownership, use or possession

of any real property situate within this Commonwealth (42 Pa.C.S. §5322(a)(1)(v)), or if the foreign

72

Page 77: FEBRUARY 2016 PENNSYLVANIA BAR … 2016 PENNSYLVANIA BAR EXAMINATION Essay Questions and Examiners’ Analyses and Performance Test Pennsylvania Board of Law Examiners 601 Commonwealth

defendant has an interest in, uses, or possesses real property in this Commonwealth (42 Pa.C.S.

§5322(a)(5)).

EEI owns real estate, and possesses real property in the Commonwealth. Complaint; Jaeger

Affidavit.

EEI’s ownership of real estate in the Commonwealth may lead to specific jurisdiction over EEI

in Pennsylvania. 42 Pa.C.S. §5322(a)(1)(v) and (a)(5).

When jurisdiction over a person is based upon section 5322, only a cause of action or other

matter arising from acts enumerated in subsection (a) or from acts forming the basis of jurisdiction under

subsection (b) may be asserted against him. 42 Pa.C.S. §5322(c); Mendel

The cause of action against EEI is the result of alleged wrongful discharge of an employee of

EEI for conduct that occurred in Oklahoma and did not arise out of EEI’s ownership of property in

Pennsylvania. Complaint. While the discharged employee’s primary duties included overseeing the

real estate in Pennsylvania, and collecting rent checks derived from the property located in

Pennsylvania, these activities occurred in Oklahoma and the cause of action did not relate to these

activities. Complaint.

While the discharged employee did travel to Pennsylvania on two occasions to deal with matters

relating to the property, i.e., signing the lease and inspecting the property for storm damage, she was

employed and discharged in Oklahoma. Thus, by her own admission, the cause of action did not arise

from EEI’s activities in Pennsylvania.

Because the cause of action is not sufficiently related to the activity that establishes jurisdiction,

specific jurisdiction does not exists in Pennsylvania under 42 Pa.C.S §5322(a).

Due Process 11 Points

There were not sufficient minimum contacts in Pennsylvania sufficient to satisfy due process.

Even if “it is determined that jurisdiction is authorized by the Long-Arm Statute, the party

seeking relief must demonstrate that the exercise of jurisdiction conforms with the Due Process Clause.

Mendel v. Williams

“Because due process may permit specific jurisdiction based solely on ‘single or occasional’ acts

purposefully directed at the forum, it is narrow in scope, limiting a cause of action to the extent that it

‘arises out of or relates to’ the very activity that establishes jurisdiction.” Mendel v. Williams

“Whether specific jurisdiction is proper under the Due Process Clause requires a two-part

analysis: first, the plaintiff must demonstrate that the defendant purposefully established minimum

contacts with the forum state; and second, the maintenance of the suit must not offend ‘traditional

notions of fair play and substantial justice.’” Mendel v. Williams

73

Page 78: FEBRUARY 2016 PENNSYLVANIA BAR … 2016 PENNSYLVANIA BAR EXAMINATION Essay Questions and Examiners’ Analyses and Performance Test Pennsylvania Board of Law Examiners 601 Commonwealth

“A defendant purposefully establishes minimum contacts with the forum state when its contacts

are: such that the defendant could reasonably anticipate being called to defend itself in the forum. . . .

Random, fortuitous, and attenuated contacts cannot reasonably notify a party that it may be called to

defend itself in a foreign forum, and, thus, cannot support the exercise of personal jurisdiction.” Mendel

v. Williams

A defendant must have purposefully directed its activities to the forum and conducted itself in a

manner indicating that it has availed itself of the forum’s privileges and benefits such that it should be

subjected to the states laws and regulations. Mendel v. Williams

Although EEI purposefully purchased the property in Pennsylvania to house its corporate

headquarters, it never occupied the building, or moved anything into the building; it tried to sell the

building, but was unable to find a buyer; the building presently has no relation to EEI’s general gas

business; and the income from the frozen yogurt company’s rental of the property totaling $14,000.00

annually is incidental to the $1 billion gas business carried on by EEI. Affidavit. Additionally, while

the discharged employee visited the property on two occasions, by her own admission in the Complaint,

there is no connection between those visits and her discharge. All these activities, even if they are found

to constitute minimum contacts, are not related to the cause of action and cannot be used as a basis for

specific jurisdiction. 42 Pa.C.S. § 5322(c).

EEI’s contacts do not make it reasonable and fair to require EEI to conduct a defense in

Pennsylvania.

Even if the defendant has purposefully established minimum contacts in the forum state, the

contacts must be considered on a case-by-case basis to determine whether they are such as to make it

reasonable and fair to require the defendant to conduct its defense in that state. Mendel v. Williams

Factors to consider when determining if it is reasonable and fair to require a defendant to conduct

its defense in a state include:

(1) the burden on the defendant;

(2) the forum state’s interest in adjudicating the dispute;

(3) the plaintiff’s interest in obtaining convenient and effective relief;

(4) the interstate judicial system’s interest in obtaining the most efficient resolution of

controversies; and

(5) the shared interest of the several states in furthering fundamental substantive social policies.

Mendel v. Williams

The burden on EEI to defend itself in Pennsylvania would be high, as the purported actions of

EEI occurred in Oklahoma; the plaintiff was employed by EEI in Oklahoma; the manager who fired the

plaintiff is in Oklahoma; and the corporate records of EEI would be maintained in Oklahoma.

Pennsylvania likely has little interest in adjudicating a claim of wrongful discharge wherein the

employee resided in Oklahoma at the time of the alleged incident; the employer at the time of the

alleged incident was, and remains, in Oklahoma; and the alleged conduct occurred in Oklahoma.

74

Page 79: FEBRUARY 2016 PENNSYLVANIA BAR … 2016 PENNSYLVANIA BAR EXAMINATION Essay Questions and Examiners’ Analyses and Performance Test Pennsylvania Board of Law Examiners 601 Commonwealth

Plaintiff will not be able to obtain convenient and effective relief in Pennsylvania because the

people she will want to depose and call at trial are out of state; and any relief (judgment) she may obtain

against EEI will require extra steps to enforce in Oklahoma.

The interstate judicial system’s interest in obtaining the most efficient resolution of this

controversy would be to adjudicate the matter in Oklahoma. Oklahoma is where the purported conduct

occurred; it is where the employee resided at the time of the purported conduct; it is where the defendant

and the majority of witnesses are located; it is where the policy relating to jury duty was allegedly

violated; and it is where the defendant’s corporate records are maintained.

While every state has an interest in protecting its citizens from wrongful discharge, an action

based thereon should be maintained in the state in which the parties resided at the time of the alleged

bad acts; where the defendant employer and its employees are located; and where the alleged incident

took place.

Based on a Due Process analysis of the facts presented, a finding of specific jurisdiction in this

matter would violate the Due Process Clause and, therefore, no specific personal jurisdiction exists over

EEI.

75